You are on page 1of 142

PAGE

.General Aptitude.

Q.1 to Q.5 carry one mark each


Q.1 Choose the appropriate word/phrase, out of the four options given below, to complete the following
sentence :
Frogs ______________
(A) croak (B) roar (C) hiss (D) patter
Ans. (A)
Q.2 If log x (5/7) = – 1/3, then the value of x is
(A) 343/125 (B) 125/343 (C) – 25/49 (D) – 49/25
Ans. (A)
a b ab
Q.3 Operates ,  and  are defined by : a  b  ; a  b ; a  b  ab .
ab a b
Find the value of (66  6)  (66  6)
(A) – 2 (B) – 1 (C) 1 (D) 2
Ans. (C)
Q.4 Choose the word most similar meaning to the given word :
Educe
(A) Exert (B) Educate (C) Extract (D) Extend
Ans. (C)
Q.5 Choose the most appropriate word from the options given below to complete the following sentence.
The principal presented the chief guest with a ___________, as token of appreciation.
(A) momento (B) memento (C) momentum (D) moment
Ans. (B)
Q.6 to Q.10 carry two marks each
Q.6 The following question presents a sentence, part of which is underlined. Beneath the sentence you find
four ways of phrasing the underlined part. Following the requirements of the standard written English,
select the answer that produces the most effective sentence.
Tuberculosis, together with its effects, ranks one of the leading causes of death in India.
(A) ranks as one of the leading causes of death
(B) rank as one of the leading causes of death
(C) has the rank of one of the leading causes of death
(D) are one of the leading causes of death
Ans. (A)
PAGE
2

Q.7 Humpty Dumpty sits on a wall every day while having lunch. The wall sometimes breaks. A person
sitting on the wall falls if the wall breaks.
Which one of the statements below is logically valid and can be inferred from the above sentences?
(A) Humpty Dumpty always falls white having lunch
(B) Humpty Dumpty does not fall sometimes while having lunch
(C) Humpty Dumpty never falls during dinner
(D) When Humpty Dumpty does not sit on the wall, the wall does not break
Ans. (B)
Q.8 A cube side 3 units is formed using a set of smaller cubes of side 1 unit. Find the proportion of the
number of faces of the smaller cubes visible to those which are NOT visible.
(A) 1 : 4 (B) 1 : 3 (C) 1 : 2 (D) 2 : 3
Ans. (C)
Q.9 Fill in the missing value

Ans. 3
Q.10 Read the following paragraph and choose the correct statement.
Climate change has reduced human security and threatened human well being. An ignored reality of
human progress is that human security largely depends upon environmental security. But on the
contrary, human progress seems contradictory to environmental security. To keep up both at the required
level is a challenge to be addressed by one and all. One of the ways to curb the climate change may be
suitable scientific innovations, while the other may be the Gandhian perspective on small scale progress
with focus on sustainability.
(A) Human progress and security are positively associated with environmental security.
(B) Human progress is contradictory to environmental security.
(C) Human security is contradictory to environmental security.
(D) Human progress depends upon environmental security.
Ans. (B)
PAGE
3

.Technical Part.

Q.1 to Q.25 carry one mark each


Q.1 A silicon sample is uniformly doped with donor type impurities with a concentration of 1016 /cm 3 . The
electron and hole mobilities in the sample are 1200cm2 /V-s and 400cm 2 /V-s respectively. Assume
complete ionization of impurities. The charge of an electron is 1.6 1019 C . The resistivity of the sample
(in  -cm) is _________.
Ans. 0.520
Sol. Given : N D  1016 /cm 3
n  1200 cm2 /V-s
 p  400 cm2 /V-s
q  1.6 1019 C
Resistivity of a semiconductor is given by,
1 1 1
  
 n q n N D q n
1

10 1.6 1019 1200
16

  0.52 -cm
Hence, the correct answer is 0.520.
Q.2 Negative feedback in a closed-loop control system DOES NOT
(A) Reduce the overall gain (B) Reduce bandwidth
(C) Improve disturbance rejection (D) Reduce sensitivity to parameter variation
Ans. (B)
Sol. Negative feedback in a closed loop
(i) Increases bandwidth
(ii) Reduces gain
(iii) Improve disturbance rejection
(iv) Reduce sensitivity to parameter variation.
Hence, the correct option is (B).
Q.3 In the network shown in the figure, all resistors are identical with R  300  . The resistance Rab (in  )
of the network is __________.

Ans. 100
PAGE
4

Sol. . Method 1 .
Since bridge is balanced hence modify the given circuit as shown below.

1 1 1 1 1
   
Req 2 R R R 2 R
1 3

Req R
R 300
Req    100 
3 3
Hence, the correct answer is 100.
. Method 2 .
Since this is balanced wheat stone bridge, modified circuit is shown below.

If n similar resistance of R ' are in parallel then we can directly calculate Req
R'
Req 
n
2 R R 300
Req     100 
6 3 3
Hence, the correct answer is 100.
PAGE
5

Q.4 In the circuit shown, at resonance, the amplitude of the sinusoidal voltage (in Volts) across the capacitor
is __________.

Ans. 25
Sol. We know that,
VC
Quality factor, Q
V
Hence, VC  QV
For series RLC circuit
1 L 1 0.1103
Q  
R C 4 1106
10
Q
4
10
Hence, VC   10 cos t
4
VC  25cos t
Amplitude of VC  25 Volt .
Hence, the correct answer is 25.
Q.5 Suppose A and B are two independent events with probabilities P( A)  0 and P( B)  0 . Let A and B be
their complements. Which one of the following statements is FALSE?
(A) P( A  B)  P( A) P( B) (B) P( A / B)  P( A)
(C) P( A  B)  P( A)  P( B) (C) P( A  B)  P( A) P( B )
Ans. (C)
Sol. Given : For two independent events A and B
We know that, P( A  B)  P( A)  P( B)
And from additive property
P( A  B)  P( A)  P( B)  P( A  B)
P( A  B)  P( A)  P( B)  P( A) P( B)
So, (C) is clearly false.
P( A  B) P( A) P( B)
P( A / B)    P( A)
P( B) P( B)
Hence, all other options are true, except (C).
PAGE
6

Q.6 In the circuit shown, the switch SW is thrown from position A to position B at time t  0 . The energy
(in μJ ) taken from the 3 V source to charge the 0.1F capacitor from 0 V to 3 V is

(A) 0.3 (B) 0.45 (C) 0.9 (D) 3


Ans. (C)
Sol. At t  0 , circuit will be in steady state

VC (0 )  VC (0 )  0 Volt


At t   ,

VC ()  3 Volt
Voltage across capacitor is given by,
VC (t )  VC ()  VC (0)  VC () et / 
Where   RC  120  0.110 6
VC (t )  3  (0  3) et /   3(1  et /  )
dVC (t ) 0.1106  3et / 
And I C (t )  C 
dt 
6 t / 
0.110  3e 1 t / 
I C (t )  6
 e
120  0.110 40
Energy taken is given by,
 
1 t / 
  VI dt   3  e dt
0 0
40

3 3
   e  t /    12  10 6
40 0 40
 0.9 J
Hence, the correct option is (C).
PAGE
7

Q.7 Let z  x  iy be a complex variable. Consider that contour integration is performed along the unit circle
in anticlockwise direction. Which one of the following statements is NOT TRUE?
z 1
(A) The residue of 2 at z  1 is (B)  z 2 dz  0
z 1 2 C

1 1
(C)
2i  C z
dz  1 (D) z (Complex conjugate of z) is analytical function

Ans. (D)
Sol. Given : z  x  iy is the complex variable.
z
(A) Function F ( z )  2
z 1
Poles of F ( z ) i.e. z 2  1  z  1
Residue at simple pole z = 1 is
z 1 1
Rz 1  lim( z  1) F ( z )  lim  
z 1 z 1 z  1 11 2
 Option (A) is true.
(B) Let F ( z)  z 2

 F ( z)dz   z dz
2
So,
C C

C is the unit circle, taken in clockwise direction.


 z 1

F ( z )  z 2 has no pole, so it is analytic everywhere, hence,

 z dz  0
2

Hence, option (B) is also true.


1 1
(C)
2i  z dz
C

1
Let F ( z) 
z
Pole, z = 0 simple pole
As z = 0 lies inside C
So,  F ( z)dz  2i R
C
z 0
PAGE
8

Residue of F ( z ) at z  0 is
Rz 0  lim z F ( z )  lim1  1
z 0 z 0

So,  F ( z)dz  2i 1


C

1
2i C
F ( z )dz  1

Hence, option (C) is also true.


(D) Let F ( z)  z
F ( z )  u  jv  x  jy  z  x  iy  x  iy 

u ( x, y )  x
v ( x, y )   y
u u
ux   1, u y  0
x y
v v
vx   0, v y   1
x y
As, ux  v y
So, Cauchy Riemann equation is not satisfied. Hence it is not an analytic function.
 Option (D) is False.
Q.8 Consider a straight, infinitely long, current carrying conductor lying on the z-axis. Which one of the
following plots (in linear scale) qualitatively represents the dependence of H  on r, where H  is the
magnitude of the azimuthal component of magnetic field outside the conductor and r is the radial
distance from the conductor?
(A) (B)

(C) (D)

Ans. (C)
Sol. A straight, infinitely long, current carrying conductor lying on the z-axis is shown below.
PAGE
9

For an infinity conductor (along z-axis), the magnetic field produced at a distance ‘r’.
I
H 
2r
or H  H  aˆ
1
H 
r

Hence, the correct option is (C).


Q.9 The waveform of a periodic signal x (t ) is shown in the figure.

 t 1 
A signal g (t ) is defined by g (t )  x   . The average power of g (t ) is _________.
 2 
Ans. 4
 t  1
g (t )  x 
 2 
Sol. Given :

Waveform of x (t ) is shown below.

E
* If x (t ) has energy of E, then energy of x(at ) will be .
a
 t  1
Energy of any signal does not change by time shifting. Hence energy of x 
 2 
* will be double of

energy of x (t ) and therefore power will also be double.


PAGE
10

Let average power of x(t )  Pav


Time period of x(t ), T  3 .
Average power Pav is given by

1
Pav  
2
x(t ) dt
T 

1
1
Pav  
2
x(t ) dt
3 2

0  2  t  1
Where x(t )  
 3t 1  t  1
1 
1 1
Pav    0  dt   (3t ) 2 dt 
3  2 1 
1
1 9  t3 
1
1
Pav   9t  dt  
2

3 1 3  3  1
1 1
Pav  3(1)3  3(1)3   3  3
3 3
Pav  2 Watt
Power of g (t )  2  power of x (t )
 2  2  4 Watt
Hence, the correct answer is 4.
Q.10 Consider a system of linear equations :
x  2 y  3 z  1 ,
x  3 y  4 z  1 , and
2 x  4 y  6 z  k
The value of k for which the system has infinitely many solution is ________.
Ans. 2
Sol. Given :
The system of linear equations,
x  2 y  3 z  1
x  3y  4z  1
 2x  4 y  6z  k
Writing in the matrix from AX  B
 1 2 3   x   1
 1 3 4   y    1 
    
 2 4  6   z   k 
PAGE
11

Also the Augmented matrix,


 1 2 3 1
 A : B    1 3 4 1 
 2 4  6 k 
and applying matrix operations, R2  R2  R1 and R3  R3  2R1
1 2 3 1 
 A : B   0 1 1 2 
0 0 0 k  2 
 Rank of matrix A, r ( A)  2
Condition to have infinite number of solutions is,
r ( A : B)  r ( A)  n
where number of unknown n = 3
So, r ( A : B)  r ( A)  2
If k 2  0
As r ( A : B )  2 there must be two non-zero rows i.e. k  2  0
 k 2 Ans.
Hence, the correct answer is 2.
Q.11 A function f ( x)  1  x 2  x3 is defined in the closed interval [ 1,1] . The value of x, in the open interval
(1,1) for which the mean value theorem is satisfied, is
1 1 1 1
(A)  (B)  (C) (D)
2 3 3 2
Ans. (B)
Q.12 A sinusoidal signal of 2 kHz frequency is applied to a delta modulator. The sampling rate and step-size
 of the delta modulator are 20,000 samples per second and 0.1 V, respectively. To prevent slope
overload, the maximum amplitude of the sinusoidal signal (in Volts) is
1 1 2
(A) (B) (C) (D) 
2  
Ans. (A)
Sol. Given : f m  2 kHz, f s  2 kHz and   0.1V
To avoid slope overload,
 dm(t )
 …(i)
TS dt max
For single-tone sinusoidal modulating signal.
m(t )  Am sin mt
dm(t )
 Amm
dt max
PAGE
12

From equation (i), we get



 Am m
TS
0.1 20 103  Am  2 2 103
1
Am 
2
1
The maximum amplitude of the sinusoidal signal is .
2
Hence, the correct option is (A).
Q.13 The electric field component of a plane wave travelling in a lossless dielectric medium is given by
 z 
E ( z , t )  aˆ y 2cos 108 t   V/m . The wavelength (in m) for the wave is ____.
 2
Ans. 8.89
 z  1
Sol. Given : E ( z , t )  a y 2 cos 108 t   V/m , Phase constant,   rad/m
 2 2

Wavelength is given by,

2 2
   2 2 m
 1/ 2

  8.89 m

Hence, the correct answer is 8.89.


Q.14 In the given circuit, the values of V1 and V2 respectively are

(A) 5 V, 25 V (B) 10 V, 30 V
(C) 15 V, 35 V (D) 0 V, 20 V
Ans. (A)
Sol. Given circuit is shown below.
PAGE
13

Applying nodal analysis at node V1


5  I  I  2I
 5  4I
Voltage is same for parallel branch
i.e. V1  4 I
 V1  5 Volt
Applying KVL in first loop,
V2  5  4  V1
V2  20  5  25 Volt
Hence, the correct option is (A).
Q.15 Consider a four bit D to A converter. The analog value corresponding to a digital signals of values 0000
and 0001 are 0 V and 0.0625 V respectively. The analog value (in Volts) corresponding to the digital
signal 1111 is __________.
Ans. (0.9375)
Q.16 In an 8085 microprocessor, the shift registers which store the result of an addition and the overflow bit
are, respectively
(A) B and F (B) A and F (C) H and F (D) A and C
Ans. (D)
Sol. Registers which store the result of an addition is accumulator. And carry flag represent the overflow.
Hence, the correct option is (D).
Q.17 A region of negative differential resistance is observed in the current voltage characteristics of a silicon
p-n junction if
(A) Both the p-region and the n-region are heavily doped
(B) The n region is heavily doped compared to the p-region
(C) The p-region is heavily doped compared to the n-region
(D) An intrinsic silicon region is inserted between the p-region and the n-region
Ans. (A)
Sol. Under normal forward bias operation as voltage begins to increase, electrons at first tunnel through the
very narrow p-n junction barrier and fill electron states in the conduction band on the n-side, which
becomes aligned with empty valence band hole states on the p-side of the p-n junction. As voltage
increases further, these states become increasingly misaligned and the current drops. This is called
negative resistance becomes current decreases with increasing voltage.
p-n diode with high doping on both sides is called tunnel diode.
Hence, the correct option is (A).
Q.18 Consider the signal s(t )  m(t )cos(2f ct )  mˆ (t )sin(2f ct ) where mˆ (t ) denotes the Hilbert transform of
m(t ) and the bandwidth of m(t ) is very small compared to f c . The signal s(t ) is a
(A) High-pass signal (B) Low-pass signal
(C) Band-pass signal (D) Double side band suppressed carrier signal
PAGE
14

Ans. (C)
Sol. Given : s(t )  m(t ) cos(2f ct )  mˆ (t )sin(2f ct ) and f c  f m
where, f c  carrier frequency
f m  modulating frequency
Let m(t )  cos 2f mt
Hilbert transform of m(t ) is given by,
ˆ (t )  cos(2fmt  900 )  sin 2f mt
m
s(t )  cos(2f mt ) cos(2f ct )  sin(2f mt )sin(2f ct )
 cos( A  B)  cos A cos B  sin A sin B 
s(t )  cos 2( f c  f m )t
The expression of s (t ) represents lower SSB and thus a bandpass signal.
Hence, the correct option is (C).
10( s  1)
Q.19 The polar plot of the transfer function G ( s )  for     will be in the
s  10
(A) First quadrant (B) Second quadrant
(C) Third quadrant (D) Fourth quadrant
Ans. (A)
10( s  1)
Sol. Given : G (s) 
s  10
10( j  1)
Put s  j , G ( j) 
( j  10)
Magnitude can be written as,
2  1
G ( j)  10
2  100
Phase angle can be written as,

G ( j)  tan 1   tan 1
10
At   0 G ( j)  1 G( j)  00
At   10 G( j)  7.106 G ( j)  39.280
At    G( j)  10 G ( j)  00

Hence, the correct option is (A).


Q.20 The result of the convolution x(t )* (t  t0 ) is
(A) x(t  t0 ) (B) x(t  t0 ) (C) x(t  t0 ) (D) x(t  t0 )
Ans. (D)
Sol. (t ) is an even function.
i.e.  (  t )   (t ) ,
PAGE
15

Hence (t  t0 )  [(t  t0 )]


 (t  t0 )
 x(t )* (t  t0 )  x(t )* (t  t0 )
 x [(t  t0 )]
 x (t  t0 )
Hence, the correct option is (D).
Q.21 A 16 Kb (  16,384 bit) memory array is designed as a square with an aspect ratio of one (number of
rows is equal to the number of columns). The minimum number of address lines needed for the row
decoder is ___________.
Ans. (7)
Q.22 For the circuit with ideal diodes shown in the figure, the shape of the output (Vout ) for the given sine
wave input (vin ) will be

(A) (B)

(C) (D)

Ans. (C)
Sol. Diode circuit can be simplified as shown below

During positive half cycle


Both diodes D1 and D2 are ON.
Hence Vout  Vin
PAGE
16

During negative half cycle


Both diodes D1 and D2 are OFF
Hence Vout  0
Hence, the correct option is (C).
1  4 1 2
Q.23 The value of p such that the vector  2  is an eigenvector of the matrix  p 2 1  is
   
 3  14  4 10 
Ans. 17
4 1 2 1 
   
Sol. Given : A   p 2 1  , X   2 
14  4 10   3 
As X is an Eigen vector of A, let the corresponding Eigen Value is  .
So, AX  X
 4 1 2  1  1 
 p 2 1   2    2
     
14  4 10  33  3  31  3 31
 12  
 p  7    2 
   
 36  31  3  31
So,   12, p  7  2 and 3  36
So, Eigen value   12
And p  7  2 12  p  17 Ans.
Hence, the correct answer is 17.
Q.24 In the circuit shown below, the Zener diode is ideal and the Zener voltage is 6 V. The output voltage V0
(in Volts) is _______.

Ans. 5
Sol. For the given regulator circuit, check whether zener diode is in breakdown or Not, by finding voltage
across its terminal.
PAGE
17

1k
Voltage V0   10
(1  1) k
V0  5 V
and VZ  6 V , hence zener diode is not in breakdown region
and output voltage V0  5 Volt .
Hence, the correct answer is 5.
K
Q.25 A unity negative feedback system has the open-loop transfer function G ( s )  . The value
s( s  1) s(3)
of gain K (  0) at which the root locus crosses the imaginary axis is ___________ .
Ans. 12
Sol. . Method 1 .
K
Given : G ( s)  , H(s) =1
s( s  1)( s  3)
The root locus branch crosses the imaginary axis is determined by applying Routh Hurwitz criterion to
the characteristic equation.
Characteristics equation is given by, 1  G ( s) H ( s)  0
K
1 0
s ( s  1) ( s  3)
s 3  4s 2  3s  K  0
Routh Tabulation :

s3 1 3
s2 4 K
12  K
s1 0  Row of zero
4
s0 K

The intersection of root locus plot with imaginary axis is given by the value of K obtained by solving the
following equation.
12  K
0  K mar  12
4
Hence, the correct answer is 12.
. Method 2 .
For third order system to be marginally stable, IP = EP
1 K  4  3
K mar  12
Hence, the correct answer is 12.
PAGE
18

Q.26 to Q.55 carry two marks each


Q.26 For the NMOSFET in the circuit shown, the threshold voltage is Vth , where Vth  0 . The source voltage
VSS is varied from 0 to VDD . Neglecting the channel length modulation, the drain current I D as a function
of VSS is represented by

(A) (B)

(C) (D)

Ans. (A)
Sol. NMOS circuit is shown below.

From above figure, VD  VG

Hence VDS  VGS

Also VDS  VGS  VT

Transistor operates in saturation region, drain current I D in saturation region is given by,

I D  k (VGS  VT )2

I D  k VDD  VSS  VT 
2
PAGE
19

As VSS increases, I D decreases

Hence, the correct option is (A).


Q.27 The longitudinal components of the magnetic field inside an air-filled rectangular waveguide made of a
perfect electric conductor is given by the following expression
H z ( x, y, z, t )  0.1cos(25x)cos(30.3 y)cos(12 109 t z)(A/m)
The cross-sectional dimensions of the waveguide are given as a = 0.08 m and b = 0.033 m. the mode of
propagation inside the waveguide is
(A) TM 12 (B) TM 21 (C) TE21 (D) TE12
Ans. (C)
Sol. Given : H z ( x, y, z, t )  0.1cos(25x)cos(30.3 y)cos(12 109 t z)(A/m)
And a = 0.08 m, b = 0.033 m
Comparing with standard magnetic field equation, we get
m
x  25x
a
m
 25
0.08
m2
n
And y  30.3y
b
n
 30.3
0.033
n 1
 The mode of propagation is TE21 .
Hence, the correct option is (C).
Q.28 The input X to Binary Symmetric Channel (BSC) shown in the figure is ‘1’ with probability of 0.8. The
1
cross-over probability is . If the received bit Y  0 , the conditional probability that ‘1’ was transmitted
7
is _______.

Ans. 0.4
PAGE
20

Sol. The binary symmetric channel is shown in figure.

P[ X  0]  0.2
P[ X  1]  0.8
According to Baye’s theorem,
P (Y  0 X  1) P( X  1)
P ( X  1 Y  0) 
P (Y  0 X  1) P( X  1)  P (Y  1 X  0) P( X  0)
1
 0.8
P [ X  1 Y  0]  7  0.4
1 1
 0.8   0.2
7 7
Hence, the correct option is 0.4.
Q.29 The transmitted signal in a GSM system is of 200 kHz bandwidth and 8 users share a common
bandwidth using TDMA. If at a given time 12 users are talking in a cell, the total bandwidth of the
signal received by the base station of the cell will be atleast (in kHz)_______.
Ans. (400)
Q.30 A lead compensator network includes a parallel combination of R and C in the feed-forward path. If the
s2
transfer function of the compensator is Gc ( s )  , the value of RC is _______.
s4
Ans. 0.5
s2
Sol. Given : Gc ( s ) 
s4
 s  s
2 1   0.5 1  
Gc ( s )  
2  2
 ….. (i)
 s   s
4 1   1  
 4  4
Transfer function for lead compensator is given by,
(1  s)
Gc ( s)  ….. (ii)
(1  s)
where   time constant
On comparing equation (i) and (ii), we get
1
   RC
2
Hence, the correct option is 0.5.
PAGE
21

Q.31 Consider a uniform plane wave with amplitude ( E0 ) of 10 V/m and 1.1 GHz frequency travelling in air,
and incident normally on a dielectric medium with complex relative permittivity ( r ) and permeability
( r ) as shown in the figure.

The magnitude of the transmitted electric field component (in V/m) after it has travelled a distance of 10
cm inside the dielectric region is __________.
Ans. 0.1
Sol. Given : E0  10 V/m , f  1.1 GHz
Medium 1 : air, 1  120  ,  r  1 , r  1
Medium 2 : dielectric, 2  120  , r  1  j 2,  r  1  j 2,   0
Since 1  2 , so E2  E1  10 V/m
Let, E3  electric field in the dielectric after travelling 10 cm
E3 will be given by,
E3  E2ez
Magnitude is reduced due to attenuation constant only.
E3  E2e z … (i)
Where   propagation constant
    j  j(  j)
    j  j  00  r  r
  j  0 0 (1  2 j )(1  2 j )
  j  00 (1  2 j )
  j  0 0  2  0 0
    j  j  0 0  2  0 0
2  21.1109
   2 00   46.07 Np/m
3 108
Form equation (i), at z  10cm,
2
E3  10e 1010 46.07
 10e 4.6
E3  0.1 V/m
Hence, the correct option is 0.1.
PAGE
22

Q.32 For a silicon diode with long P and N regions, the accepter and donor impurity concentrations are
11017 cm 3 and 11015 cm 3 , respectively. The lifetimes of electrons in P region and holes in N region
are both 100 s . The electron and hole diffusion coefficient are 49cm 2 /s and 36cm 2 /s , respectively.
Assume kT / q  26 mV , the intrinsic carrier concentration is 11010 cm 3 , and q  1.6 1019 C . When a
forward voltage of 208 mV is applied across the diode, the hole current density (in nA/cm 2 ) injected
from P region to N region is _______.
Ans. 286.17
Sol. The hole current density injected from p region to n region is given by,
q ni2 D p   VFBT  
V 

Jp  e  1 …..(i)
N D Lp  
 
Where q = charge on electron
ni  intrinsic carrier concentration in silicon
N D  Donor doping
D p  hole diffusion constant
L p  Mean diffusion length of hole
VFB  Forward voltage applied across diode
KT
VT   26 mV
q
L p   p D p  100  106  36
L p  0.06 cm
Putting all the value in equation (i), we get
1.6 1019  (1010 ) 2  36  208 
Jp    e 26
 1 
1015  6 102  
J p  28.6 109 A/cm2  28.6 nA/cm2
Hence, the correct answer is 286.17.
 K  1
Q.33 A plant transfer function is given as G ( s )   K P  I  . When the plant operates in a unity
 s  s ( s  2)
feedback configuration, the condition for the stability of the closed loop system is
K
(A) K P  I  0 (B) 2 K I  K P  0 (C) 2 K I  K P (D) 2 K I  K P
2
Ans. (A)
( sK  K )
Sol. Given : G ( s)  3 P 2I
( s  2s )
Characteristics equation is given by, 1  G ( s) H ( s)  0
s3  2s 2  s1K p  K I  0
PAGE
23

Routh Tabulation :

s3 1 KP 0
s2 2 KI 0
(2 K P  K I )
s1 0 0
2
s0 KI 0 0

For the system to be stable, all the roots must be in the left-half of s-plane, and thus, all the coefficients
in the first column of Routh's tabulation must have the same sign. Therefore, first column of the Routh’s
array should be positive.
KI  0
 2K P  K I 
 0
 2 
K
KP  I  0
2
Hence, the correct option is (A).
Q.34 Which one of the following graphs describes the function f ( x)  e x ( x 2  x  1) ?

(A) (B)

(C) (D)

Ans. (B)
Q.35 A 3-input majority gate is defined by the logic function M (a, b, c)  ab  bc  ca . Which one of the
following gate is represented by the function M (M (a, b, c), M (a, b, c )c) ?
(A) 3-input NAND gate (B) 3-input XOR gate
(C) 3-input NOR gate (D) 3-input XNOR gate
Ans. (B)
Sol. . Method 1 .
Given : M (a, b, c)  ab  bc  ca
We have to find out
PAGE
24

M  M (a, b, c ), M (a, b, c )c 

M (a, b, c)  (ab  bc  ca)


M (a, b, c )  ab  bc  ca

Then M  M (a, b, c), M (a, b, c ), c   (ab  bc  ca)(ab  bc  ca) (ab  bc  ca)c  c(ab  bc  ca)

 (ab.bc.ca ) ( ab  bc  ca )  (ab  bc  ca ).c  (ab.bc.ca ).c 

 (a  b ) (b  c )(c  a )  (ab  bc  ca) abc (a  b )(b  c )(c  a ).c 

 (a  b )(b  c )(c  a )[ab  bc  ca  c] abc


 (a  b )(b  c )(c  a )(a  b  c)  abc
 (ab  bc  ca )(a  b  c)  abc
 a b c  b c a  ca b  abc
F  a b c
Hence, the correct option is (B).
. Method - 2 .
M (a, b, c)  ab  bc  ca
K-map for M (a, b, c) is shown below

M (a, b, c)  m(3,5, 6, 7)
Hence M ( a, b, c )  m(3, 5, 6, 7)  m(0,1, 2, 4)
Now M (a, b, c)  ab  bc  ca
K-map for M (a, b, c ) is shown below

M (a, b, c )  m(2, 4, 6, 7)
Now M ( a , b, c )  c
K-map is shown below
PAGE
25

 m(1,3,5, 7)

Hence M  M (a, b, c), M (a, b, c ).c 


 m(0,1, 2, 4)m(2, 4, 6, 7)  m(2, 4, 6, 7)m(1,3,5, 7)  m(1,3,5, 7)m(0,1, 2, 4)
 m(2, 4)  m(7)  m(1)
 m(1, 2, 4, 7)
 a b c
Hence, the correct option is (B).
Q.36 In the system shown in figure (a), m(t) is a low-pass signal with bandwidth W Hz. The frequency
response of the band-pass filter H(f ) is shown in figure (b). If it is desired that the output signal
z (t )  10 x(t ) , the maximum value of W (in Hz) should be strictly less than ________.

Fig. (a)

Fig. (b)
Ans. 350
Sol. We have the input signal,
x(t )  m(t ) cos(2400t )
x(t )  m(t ) cos(t )
and f  1200 Hz
So, y (t )  10 x(t )  x 2 (t )
y (t )  10m(t ) cos(2400t )  m2 (t ) cos 2 (2400t )
 cos(2t )  1 
y (t )  10m(t ) cos(t )  m 2 (t )  
 2
m2 (t ) m2 (t ) cos 2t
y (t )   10m(t ) cos t 
2 2
m(t ) is a low pass signal with bandwidth W then m 2 (t ) will have bandwidth 2W .
Second term in the above equation represents DSB-SC signal centred at 1200 Hz and bandwidth 2W.
Third term in the above equation represents DSB-SC signal centred at 2400 Hz and bandwidth 4W.
Spectrum of y (t ) is shown in figure below.
PAGE
26

From the frequency plot, we conclude the following results.


Result 1 1200 W  700
W  500
Result 2 1200  W  1700
W  500
Result 3 1200 W  2W
1200  3W
W  400
Result 4 2400  2W  1700
2400 1700  2W
2W  700
W  350
Thus, the above conclusions result in
W  350
Hence, the correct answer is 350.
Q.37 A MOSFET in saturation has a drain current of 1 mA for VDS  0.5V . If the channel length modulation
coefficient is 0.05V 1 , the output resistance (in kΩ ) of the MOSFET is ________.
Ans. 20
Sol. Given : Drain current I D  1 mA at VDS  0.5 V .
Channel length modulation coefficient   0.05 V 1
Under channel length modulation
I D  I Dsat (1  VDS )
dI D 1
  I D sat
dVDS r0
1 1
r0    20 k
 I D sat 0.05 103
Hence, the correct answer is 20.
Q.38 For the discrete-time system shown in the figure, the poles of the system transfer function are located at
PAGE
27

1 1 1 1
(A) 2, 3 (B) ,3 (C) , (D) 2,
2 2 3 3
Ans. (C)
Sol.

1 5
y  n  x  n  y  n  2  y  n  1
6 6
5 1
y  n 
y  n  1  y  n  2  x  n 
6 6
Taking z-transform on both sides, we get
5 1
Y ( z )  z 1Y ( z )  z 2Y ( z )  X ( z )
6 6
 5 1 
Y ( z ) 1  z 1  z 2   X ( z )
 6 6 
Y ( z) 1
 H ( z) 
X ( z)  5 1 1 2 
1  z  z 
 6 6 
z2 z2
H ( z)  
 5 5 1  1  1
 z  z    z   z  
 6 6  2  3
For poles of the system
 1  1
 z   z    0
 2  3
1 1
z ,
2 3
PAGE
28

1 1
Poles of the system  ,
2 3
Hence, the correct option is (C).
Q.39 A vector P is given by P  x 3 yax  x 2 y 2a y  x 2 yzaz . Which of the following statements is TRUE?
(A) P is solenoidal, but not irrotational (B) P is irrotational, but not solenoidal
(C) P is neither solenoidal nor irrotational (D) P is both solenoidal and irrotational
Ans. (A)
Sol. Given : P  x3 yax  x 2 y 2 a y  x 2 yzaz
For solenoidal,   P  0
P Py P
 P  x  
x y z
 P  3x2 y  2x2 y  x2 y  0
Therefore, P is solenoidal.
For irrotational,  P  0
ax ay az
  
 P   ax ( x 2 z )  a y (2 xyz )  az (2 xy 2  x3 )  0
x y z
x y  x2 y 2
3
 x 2 yz
Therefore, P is not irrotational.
Hence, the correct option is (A).
Q.40 In the circuit shown, assume that the opamp is ideal. The bridge output voltage V0 (in mV) for   0.05
is______.

Ans. 250
Sol. Given circuit is shown below.
PAGE
29

Due to virtual ground concept


V  V  1V
Current in 100  and 50  resistor will be same
1
I 50   I100   A
50
So, output voltage is given by,
1 1
V0   250(1  )  250(1  )   500
100 100
V0  5   0.05  0.25 V
V0  250 mV
Hence, the correct answer is 250.
Q.41 The circuit shown in the figure has an ideal opamp. The oscillation frequency and the condition to
sustain the oscillations, respectively, are

1 1
(A) and R1  R2 (B) and R1  4 R2
CR CR
1 1
(C) and R1  R2 (D) and R1  4 R2
2CR 2CR
Ans. (D)
Sol. Circuit is shown below
PAGE
30

Above circuit is a Wein bridge oscillator frequency of oscillation is given by


1
0  [because time constant is doubled hence frequency becomes half]
2RC
Feedback voltage V f is given by
z2
Vf  V0
z1  z2
1
Where z1  2 R 
jc
1
R
j C R
and z2  
R
1 j 2RC  1
2 j C
Vf z2
Hence 
V0 z1  z2
R
Vf j 2RC  1
 
V0 R 1
 2R 
j 2RC  1 j C
Vf jRC

V0 R( jC )  2 R( jC )(2 jRC  1)  (2 jRC  1)
Vf jRC

V0 jRC  2 R(2 RC 2  jC )  1  2 jRC
2

Vf jRC

V0 jRC  4 R C  jRC  2 jRC  1
2 2 2

Vf jRC

V0 (1  4 R 2C 2 )  j 5RC
2
PAGE
31

Feedback factor should not contain imaginary part for oscillation.


It is possible when
1  42 R 2C 2  0
1
2  2 2
4R C
1
 rad/sec
2RC
Vf jRC 1
and   
V0 5 jRC 5
R1
gain of the amplifier A  1 
R2
according to Brakhausen criteria
A  1
For limiting case A  1
 R1 
1     1
 R2 
 R1  1
1     5
 R2  
R1
4
R2
R1  4 R2
Hence, the correct option is (D).
Q.42 In the circuit shown, I1  80 mA and I 2  4mA . Transistors T1 and T2 are identical. Assume that the
thermal voltage VT is 26 mV at 27 0 C . At 500 C , the value of voltage V12  V1  V2 (in mV) is

Ans. 83.85
Sol. Given : I1  80 mA , I 2  4 mA
VT  26 mV at 27 0 C
Circuit is shown below
PAGE
32

Current I 2 is given by
 VBVE2 
I 2  I s  e T  1
 
 
VB E2

or I 2  I s e VT
where VBE2  V2
current I1 is given by
VBE1

I1  I s e VT
1V V
2
I1
Where  e VT
I2
V1 V2
80
 e VT
4
V1 V2

20  e VT
Since VT at 27 0 C is 26 mV then VT at 500 C is 27.99 mV
Thus V1  V2  VT ln(20)
Take   1 , V1  V2  27.99 ln(20)
V1  V2  83.85 mV
Hence, the correct answer is 83.85.
Q.43 The built-in potential of an abrupt p-n junction is 0.75 V. If its junction capacitance (CJ ) at a reverse
bias (VR ) of 1.25 V is 5 pF, the value of CJ (in pF) when VR  7.25V is____.
Ans. 2.5
Sol. Given : Built in potential Vbi  0.75 V
VR1  1.25 V VR 2  7.25 V
Junction capacitance, C j  5 pF
We know that
PAGE
33

1
Cj 
(Vbi  VR ) n
1
n For abrupt junction
2
(C j )1 Vbi  VR 2

(C j )2 Vbi  VR1
5 0.75  7.25

(C j ) 2 0.75  1.25
(C j )2  2.5 pF
Hence, the correct answer is 2.5.
Q.44 The electric field intensity of a plane wave traveling in free space is given by the following expression,
E( x, t )  ay 24  cos(t  k0 x)(V/m)
In this field, consider a square area 10cm 10cm on a plane x  y  1 . The total time-averaged power (in
mW) passing through the square area is______.
Ans. 53.29
Sol. Given : E ( x, t )  a y 24 cos(t  k0 x) V/m
Medium : Free space
Time average power density vector is given by,
E02
Pav  ak W/m2
2
Where, E0  24 V/m,   Intrinsic impedance of medium
  120 
(24) 2
Pav  ax W/m 2
2 120
Pav  7.53 ax W/m2
Power, P   Pav ds
s

Given plane, x  y  1
Scalar point function for given plane will be,   x  y  1
Unit vector normal to this surface will be given by,

an 

ax  a y
an 
2
ax  a y
 unit vector normal to the surface
2
PAGE
34

a 
 P   (7.53 ax ).  x  dy dz
s
 2
7.53
P 10 10 10 2 10 2
2
P  53.29 10 3 W  53.29 mW Ans.
Hence, the correct answer is 53.29.
K ( s  4)
Q.45 The open-loop transfer function of a unity feedback configuration is given as G ( s )  . The
( s  8)( s 2  9)
value of a gain K (  0) for which 1  j 2 lies on the root locus is _____.
Ans. 25.54
Sol. Method 1
Pole-zero diagram is shown below.

Vector Lengths :
AP  (7)2  (2)2  53
BP  (3)2  (2)2  13
CP  (2)2  (2)2  8
DP  (4)2  (2)2  20
Product of vector lengths drawn from the poles of G ( s ) H ( s ) to P
K
Product of vector lengths drawn from the zeros of G ( s ) H ( s ) to P
53  8  20
K  25.5
13
Hence, the correct option is 25.5.

Method 2
By magnitude criteria,
G(s) H (s) s 12 j  1
K  ( 1  2 j )  4 
G ( s ) s 1 2 j 
 (1  2 j )  8 (1  2 j ) 2  9 
PAGE
35

K 2 j 3
1
7 2 j 2 2 j 4 2 j
20 8 53
K  25.5
13
Hence, the correct option is 25.5.
Q.46 In the circuit shown, switch SW is closed at t  0 . Assuming zero initial conditions, the value of Vc (t )
(in Volts) at t  1sec is _____.

Ans. 2.528
Sol. At t  0 , circuit will be in steady state.
Hence, capacitor will acts as open circuit

VC (0 )  0 V  VC (0 )
At t   , circuit will be in steady state

2
VC ()  10  4 Volt
23
Voltage across capacitor is given by,
VC (t )  VC ()  VC (0 )  VC ()  et / 
VC (t )  4  0  4 et / 
VC (t )  4 (1  e  t /  )
3 2 6
Req  3 || 2    1.2 
3 2 5
Time constant is given by,
5
  Req  C  1.2   1 sec
6
Hence, VC (t )  4(1  e  t )
PAGE
36

Putting t  1 sec, we get


 1
VC (1)  4(1  e 1 )  4 1  
 e
VC (1)  2.528 Volt
Hence, the correct answer is 2.528.
d2y dy
Q.47 The solution of the differential equation 2
 2  y  0 with y (0)  y '(0)  1 is
dt dt
(A) (2  t )et (B) (1  2t )e  t (C) (2  t )et (D) (1  2t )et
Ans. (B)
Sol. Given : The differential equation
d2y dy
2
2  y 0
dt dt
Also, y (0)  y '(0)  1
The differential equation can be written as,
( D 2  2 D  1) y  0
d
where D
dt
Auxiliary equation is,
m 2  2m  1  0
(m  1)2  0
m  1,  1
 Solution is,
y  (C1  C2t ) e  t
Putting t  0, y (0)  (C1  0) 1
1  C1
And y '   (C1  C2t )et  C2et
 (C2  C1 )e  t  C2te  t
For t  0 , y '(0)  (C2  C1 ) 1
1  C2  C1
C2  C1  1
As, C1  1  C2  2
 t
y (t )  (1  2t )e
Hence, the correct option is (B).
Alternate Method :
Taking Laplace transform of both sides,
d2y dy 
L  2  2  y  0
 dt dt 
PAGE
37

 s 2Y (s)  sy(0)  y '(0)   2  sY (s)  y(0)  Y (s)  0


( s 2  2s  1)Y ( s)  s 1  1  2 1  0
s3 s3
Y ( s)  2 
s  2s  1 ( s  1) 2
Taking inverse Laplace transform of both sides,
 ( s  1)  2  t 1  s  2 
y (t )  L1  2 
e L  2 
 ( s  1)   s 
 1   1 
y (t )  e t  L1    2 L1  2    (1  2t )e  t
 s  s 
Hence, the correct option is (B).
Q.48 The maximum area (in square unit) of a rectangle whose vertices lies on the ellipse x 2  4 y 2  1 is
______
Ans. 1
Sol. Given : An ellipse x 2  4 y 2  1

For the ellipse shown x 2  4 y 2  1 , the rectangle has been represented inside for point P ( x, y ) i.e. one
of the four corners of rectangle lying on ellipse. The area of rectangle in first quadrant = xy
 Area of rectangle is,
1  x2
A  4 xy  4 x  4x 1  x2
4
For maximum area
dA
Equating 0
dx
dA  1 
 2 1 1  x 2  x (1  x 2 ) 1/2 ( 2 x)   0
dx  2 
x2
1  x2  0
1  x2
(1  x 2 )  x 2  0
1  2 x2  0
1
x
2
PAGE
38

1 1
Amax  2 x 1  x 2  2  1  1 Ans.
2 2
Hence, the correct answer is 1.
Q.49 The Boolean expression F ( X , Y , Z )  XYZ  XY Z  XY Z  XYZ converted into canonical product of
sum (POS) form is
(A) ( X  Y  Z )( X  Y  Z )( X  Y  Z )( X  Y  Z )
(B) ( X  Y  Z )( X  Y  Z )( X  Y  Z )( X  Y  Z )
(C) ( X  Y  Z )( X  Y  Z )( X  Y  Z )( X  Y  Z )
(D) ( X  Y  Z )( X  Y  Z )( X  Y  Z )( X  Y  Z )
Ans. (A)
Sol. . Method - 1 .
Given : F ( XYZ )  X Y Z  X Y Z  X Y Z  XYZ
K-map for function F ( XYZ ) is shown below

K-map corresponding to POS form is shown below

Hence F ( XYZ )  ( X  Y )( X  Z )(Y  Z )


If we take each maxterm separately then
F ( XYZ )  ( X  Y  Z )( X  Y  Z ) ( X  Y  Z )( X  Y  Z )
Hence, the correct option is (A).
. Method - 2 .
The Boolean expression is
F ( X , Y , Z )  X Y Z  X Y Z  X Y Z  XYZ
  m (2, 4, 6,7)
 m(0,1,3,5)
 ( X  Y  Z )( X  Y  Z )( X  Y  Z ) ( X  Y  Z )
Hence, the correct option is (A).
Q.50 The damping ratio of a series RLC circuit can be expressed as
R 2C 2L R C 2 L
(A) (B) 2 (C) (D)
2L RC 2 L R C
Ans. (C)
PAGE
39

Sol. Series RLC circuit is shown in figure.

Applying voltage division rule,


Vi ( s )
V0 ( s )  Cs
 1 
 R  Ls  
 Cs 
1
V0 ( s ) 1 LC
  …. (i)
Vi ( s )  1  R 1
Cs  R  Ls   s  s 
2

 Cs  L LC
Transfer function for second-order system is given by,
2n
T (s)  2 …. (ii)
s  2n s  n2
On comparing equation (i) and (ii), we get
1 1
2n   n 
LC LC
R
2n 
L
R R C
 LC 
2L 2 L
Hence, the correct option is (C).
Q.51 The pole-zero diagram of a causal and stable discrete-time system is shown in the figure. The zero at the
origin has multiplicity 4. The impulse response of the system is h [ n ] . If h [0]  1 , we can conclude

(A) h [ n ] is real for all n (B) h [ n ] is purely imaginary for all n


(C) h [ n ] is real for only even n (D) h [ n ] is purely imaginary for only odd n
Ans. (A)
Sol. Pole-zero diagram is shown below.
PAGE
40

1 1 1 1
Poles are s  j , j
2 2 2 2
Therefore transfer function H ( z ) is given by,
z4
H ( z) 
 1  1  
2
1  1
2

 z      z    
 2  4   2  4 
z4
H ( z) 
 2 1  2 1
 z  z   z  z  
 2  2
4
z
H ( z) 
1
z4  z2   z2
4
4
z
H ( z) 
1
z4 
4

1
H ( z)  1  z  4 
4
Taking inverse z-transform
1
h  n     n     n  4 
4
Therefore h  n is real for all n.
Hence, the correct option is (A).
Q.52 Two sequences [a, b, c ] and [ A, B, C ] are related as,
PAGE
41

 A  1 1 1  a  2
 B   1 W 1 W 2   b  where W3  e
j
3
   3 3  

C  1 W32


W34   c 
If another sequence [ p, q, r ] is derived as,
 p  1 1 1  1 0 0   A / 3
 q   1 W 1 W 2  0 W 2 0   B / 3
   3 3  3  
 r  1 W3 W3  0 0 W34  C / 3
2 4

then the relationship between the sequence [ p, q, r ] and [a, b, c ] is


(A) [ p, q, r ]  [b, a, c] (B) [ p, q, r ]  [b, c, a]
(C) [ p, q, r ]  [c, a, b] (D) [ p, q, r ]  [c, b, a]
Ans. (C)
 A  1 1 1  a
Sol. Given :  B   1 W 1 W 2   b 
   3 3  

C  1 W32


W34   c 

 A  a  b  c 
 B    a  bW 1  cW 2 
   3 3 

C   a  bW3  cW31 


2

Another sequence  p, q, r  is given by

 p  1 1 1  1 0 0   A/3
 q   1 W 1 W 2   0 W 2 0   B /3
   3 3  3  
 r  1 W32 W34   0 0 W34  C /3

 abc 
 3 
 p  1 W3 W34   
2
1 2
 q   1 W 1W 2 2 4   a  bW3  cW3 
   3 3 W3 W3   
3
 r  1 W32W32 W34W34   2 1

 a  bW3  cW3 
 3 
2
j
Where W31  e 3

4
j
W e
3
2 3

2
j
1
W3 e 3

4
j
W32  e 3
PAGE
42

2
j
W34  W31  e 3

 p c 
Therefore,  q    a 
   
 r   b 
Hence, the correct option is (C).
Q.53 All the logic gates shown in the figure have a propagation delay of 20 ns. Let A  C  0 and B  1 unit
time t  0 . At t  0 , all the inputs flip (i.e. A  C  1 and B  0 )and remain in that state. For t  0 ,
output Z  1 for a duration (in ns) of

Ans. (40)
Sol. Given logic circuit is shown below

From above figure


X B
Y  A.X
Z  Y C
Z  A.X  C
Z  A.B  C
PAGE
43

Output Z  1 is for duration of 40ns.


Hence, the correct answer is 40.

Q.54 In the given circuit, the maximum power (in Watts) that can be transferred to the load RL is

Ans. 1.65
Sol. Thevenin’s equivalent across RL i.e. across ab.

400 2
Current, I   2  450
2  j 2 (1  j )
Vth  j 2I  j 2  2  450

Vth  2 2450 V
PAGE
44

Thevenin’s impedance Z th across ab.

2  j2 j2
Zth  2 || ( j 2)  
2  j 2 (1  j )
2900
Zth   2450
245 0

For maximum power transfer to RL


RL  Z th  2 
Load current is given by,
Vth 2 2450
IL  
RL  Zth 2  2450
2450 2450
IL  
1  1450 1  1  j 1
2 2
2450
IL   1.0822.40 A
1.8522.6 0

I L  1.08
Pmax  I L2 RL  (1.08)2  2  1.65 W
Hence, the correct answer is 1.65.
1 2
Q.55 A source emits bit 0 with probability and bit with probability . The emitted bits are communicated
3 3
to the receiver. The receiver decides for either 0 or 1 based on the received value R. It is given that the
conditional density functions of R as
1 1
 , 3  x  1  , 1  x  5
f R|0 (r )   4 and f R|0 (r )   6

 0, Otherwise 
 0, Otherwise
PAGE
45

The minimum decision error probability is


1 1 1
(A) 0 (B) (C) (D)
12 9 6
Ans. (D)
Sol. Given the conditional density function of R as
1
 , 3  r  1
f R /0 (r )   4
0, otherwise
1
 , 1  r  5
f R /1 (r )   6

0, otherwise
Decision error probability that receiver decides 0 for a transmitted bit 1 is
1
f R /1 (r  0) 
6
Again, the decision error probability that receiver decides 1 for a transmitted bit 0 is
1
f R /0 (r  1) 
4
Hence, the minimum decision error probability is
1
f R /1 (r  0) 
6
Hence, the correct option is (D).


PAGE
1

.General Aptitude.

Q.1 to Q.5 carry one mark each


Q.1 Choose the word most similar in meaning to the given word :
Awkward
(A) Inept (B) Graceful (C) Suitable (D) Dreadful
Ans. (A)
Q.2 Choose the appropriate word/phase, out of the four options given below, to complete the following
sentence :
Dhoni, as well as the other team members of the Indian team_____present on the occasion.
(A) were (B) was (C) has (D) have
Ans. (C)
Q.3 An electric bus has onboard instruments that report the total electricity consumed since the start of the
trip as well as the total distance covered. During a single day of operation, the bus travels on stretches
M, N, O and P, in that order. The cumulative distance travelled and the corresponding electricity
consumption are shown in the Table below :
Stretch Cumulative distance (km) Electricity used (kWh)
M 20 12
N 45 25
O 75 45
P 100 57
The stretch where the electricity consumption per km is minimum is
(A) M (B) N (C) O (D) P
Ans. (D)
Q.4 What is the adverb for the given word below?
Misogynous
(A) Misogynousness (B) Misogynity (C) Misogynously (D) Misogynous
Ans. (C)
Q.5 Ram and Ramesh appeared in an interview for two vacancies in the same department. The probability of
Ram’s selection is 1/6 and that of Ramesh is 1/8. What is the probability that only one of them will be
selected?
(A) 47/48 (B) ¼ (C) 13/48 (D) 35/48
Ans. (B)
Q.6 to Q.10 carry two marks each
Q.6 Given below are two statements followed by two conclusions. Assuming these statements to be true,
decide which one logically follows.
Statements :
I. All film star are playback singers.
II. All film directors are film stars.
PAGE
2

Conclusions :
I. All film directors are playback singers.
II. Some film stars are film directors.
(A) Only conclusion I follows. (B) Only conclusion I nor II follows.
(C) Neither conclusion I nor II follows. (D) Both conclusions I and II follow.
Ans. (D)
Q.7 In the following sentence certain parts are underlined and marked P, Q and R. One of the parts may
contain certain error or may not be acceptable in standard written communication. Select the part
containing an error. Choose D as your answer if there is no error.
The student concreted all the errors that the instructor marked on the answer book.
(A) P (B) Q (C) R (D) No Error
Ans. (B)
Q.8 If a 2  b 2  c  1 , then ab  bc  ac lies in the interval
(A) [1, 2/3] (B) [–1/2, 1] (C) [–1, 1/2] (D) [2, –4]
Ans. (B)
Q.9 Lamenting the gradual sidelining of the arts in school curricula, a group of prominent artists wrote to the
Chief Minister last year, asking him to allocate more funds to support arts education in schools.
However, no such increase has been announced in this year’s Budget. The artists expressed their deep
anguish at their request not being approved, but many of them remain optimistic about finding in the
future.
Which of the statement(s) below is/are logically valid and can be inferred from the above statements?
(i) The artists expected funding for the arts to increase this year.
(ii) The Chief Minister was receptive to the idea of increasing funding for the arts.
(iii)The Chief Minister is a prominent artists.
(iv) Schools are giving less importance to arts education nowadays.
(A) (iii) and (iv) (B) (i) and (iv) (C) (i), (ii) and (iv) (D) (i) and (iii)
Ans. (B)
Q.10 A tiger is 50 leaps of its own behind a deer. The tiger takes 5 leaps per minute to the deer’s 4. If the tiger
and the deer cover 8 metre and 5 metre per leap respectively, what distance in metres will the tiger have
to run before it catches the deer?
Ans. 800
PAGE
3

.Technical Part.

Q.1 to Q.25 carry one mark each


dy 1  cos 2 y
Q.1 The general solution of the differential equation  is
dx 1  cos 2 x
(A) tan y  cot x  c (c is a constant) (B) tan x  cot y  c (c is a constant)
(C) tan y  cot x  c (c is a constant) (D) tan x  cot y  c (c is a constant)
Ans. (C)
dy 1  cos 2 y
Sol. Given : The differential equation is 
dx 1  cos 2 x
Integrating on both sides
dy 1  cos 2 y
 dx   1  cos 2 x
dy dx
 1  cos 2 y   1  cos 2 x
dy dx  1  cos 2 A 1  cos 2 A 
 2 cos2 y   2sin 2 x  2
 cos 2 A and
2
 sin 2
A


 sec y dy   cosec2 x dx  sec A dA  tan A and  cosec A dA  cot A
2 2 2

tan y   cot x  c
cot x  tan y  c
Hence the correct option is (C).
n
Q.2 Two causal discrete-time signals x[n] and y[ n ] are related as y[n]   x[m] . If the z-transform of
m0
y[ n ]

2
is , the value of x[2] is _________.
z ( z  1) 2
Ans. 0
n
Sol. Given : y  n   x m
m0

Taking z-transform on both sides, we get


 n 
Z  y  n   Z   x  m 
 m0 
1
Y ( z)   X ( z)
1  z 1
2
Also Y ( z) 
z ( z  1) 2
2 z
Therefore   X ( z)
z ( z  1) 2
( z  1)
2 2
X ( z)  2  z 2 
z ( z  1) ( z  1)
PAGE
4

z
X ( z )  2 z 3 
( z  1)
Taking inverse z-transform on both sides, we get
x  n  2u  n  3
Put n = 2, x  2  2u  2  3  2u  1  0
Hence, the correct answer is 0.
K
Q.3 A unity negative feedback system has an open-loop transfer function G ( s)  . The gain K for
s( s  10)
the system to have a damping ratio of 0.25 is _________.
Ans. 400
K
Sol. Given : G (s)  2 ,   0.25
s  10 s
Closed-loop transfer function is given by,
G(s)
T (s) 
1  G( s)
K
T (s)  2 …. (i)
s  10s  K
Transfer function for second-order system is given by,
2n
T (s)  2 …. (ii)
s  2n s  n2
On comparing equation (i) and (ii), we get
K  2n  n  K
2n  10
10
  0.25
2 K
10
K   20
0.5
K  (20)2  400
Hence, the correct answer is 400.
C ( s)
Q.4 For the signal flow graph shown in the figure, the value of is
R( s)
PAGE
5

G1G2G3G4
(A)
1  G1G2 H1  G3G4 H 2  G2G3 H 3  G1G2G3G4 H 1H 2
G1G2G3G4
(B)
1  G1G2 H1  G3G4 H 2  G2G3 H 3  G1G2G3G4 H 1H 2
1
(C)
1  G1G2 H1  G3G4 H 2  G2G3 H 3  G1G2G3G4 H 1H 2
1
(D)
1  G1G2 H1  G3G4 H 2  G2G3 H 3  G1G2G3G4 H 1H 2
Ans. (B)
Sol. The given signal flow graph is shown below.

Forward path : P1  G1G2G3G4


Individual loops : L1  G1G2 H1 , L2   G3G4 H 2 , L3   G2G3 H 3
Two non-touching loops : L1 L2  G1G2G3G4 H1H 2
Determinant :   1  ( L1  L2  L3 )  L1L2
 1  G1G2 H1  G3G4 H 2  G2G3 H 3  G1G2G3G4 H1H 2
Path factor : All the loops touches forward path
1  1  0  1
Using Mason’s gain formula transfer function can be written as,
C ( s) 1
  Pk  k
R( s )  k
So, the transfer function is
P
T (s)  1 1

G1G2G3G4

1  G1G2 H1  G3G4 H 2  G2G3 H 3  G1G2G3G4 H1H 2
Hence, the correct option is (B).
16 3
Q.5 A piece of silicon is doped uniformly with phosphorous with a doping concentration of 10 /cm . The
expected value of mobility versus doping concentration for silicon assuming full dopant ionization is
19
shown below. The charge of an electron is 1.6 10 C . The conductivity (in S cm-1 ) of the silicon at
300 K is ________.
PAGE
6

Ans. 1.92
Sol. From the given graph
At concentration 1016 /cm3 (1.E+16) the mobility of electron n  1200 cm2 /V-s
 Conductivity  n q  n  N D q  n
 1016 1.6 1019 1200
 1.92 Scm1
Hence, the correct answer is 1.92.
Q.6 The magnitude and phase of the complex Fourier series coefficient ak of a periodic signal x (t ) are shown
in the figure. Choose the correct statement from the four choices given. Notation : C is the set of
complex number, R is the set of purely real numbers, and P is the set of purely imaginary numbers.

(A) x (t )  R
(B) x (t )  P
(C) x(t )(C  R )
(D) the information given is not sufficient to draw any conclusion about x (t )
PAGE
7

Ans. (A)
Sol. From the phase plot
ak  00 or 180
0

Fourier series coefficient ak is given by


ak  ak e jak
ak  ak which is purely real.
Now ak is purely real if and only if x (t ) is purely real and symmetric.
Hence, the correct option is (A).
Q.7 In the circuit shown, V0  V0 A for switch SW in position A and V0  V0B for SW in position B. Assume
V
that the opamp is ideal. The value of 0 B is ______.
V0 A

Ans. 1.5
Sol. Given circuit is shown below.

When switch is in position A,


PAGE
8

Applying nodal analysis at inverting terminal, we get


Va  5 Va  V0A
 0
1 k 1 k
V0A  2Va  5
1
Where Va  1  0.5 V
1 1
 V0 A  2  0.5  5   4 V
When switch is at position B

Non-inverting input Va  0
Applying nodal analysis at inverting terminal
Va  1 Va  5 Va  V0 B
  0
1 k 1 k  1 k
1  5  V0 B  0
V0 B   6 V
V0 B  6
Therefore,   1.5
V0 A  4
Hence, the correct answer is 1.5.
Q.8 In a source free region in vaccum, if the electrostatic potential   2x 2  y 2  cz 2 , the value of constant c
must be _________.
Ans. (–3)
Sol. Given :   2x 2  y 2  cz 2
PAGE
9

In source-free region,
 D  0
( E)  0
 E  0
Where E  V  
  
Now V  ax  a y  az
x y z
 E  4 xax  2 ya y  2czaz
Again   E  0
Ex E y Et
   4  2  2c  0
x y z
c  3
Hence, the correct answer is –3.
Q.9 In the circuit shown, the average value of the voltage Vab (in Volts) in steady state condition is ____.

Ans. 5
Sol. In the circuit, 2 sources are present, hence applying super position theorem.
Considering only DC source :
In steady state capacitor will be open circuited and inductor will be short circuited.

Hence, Vab  5 V
Consider only AC source :

Steady state voltage Vab can be determined using phasor analysis


vs  5 sin(5000t )
5
Rms Vs 
2
PAGE
10

5
Vs  Vs 00  00  11.1100 V
2
Z L  jL  j5000 1103  j5 
1 1
ZC     j 200 
jC j 5000 106
11.1100 11.1100
I 
3000  j5  j 200 3000  j195
I  0.00373.720 A
So, Vab   I  ZC
  0.00373.72  ( j 200)
 0.7493.720 V
Rms Vab  0.74 V
Peak Vm  0.74 2  1.05 V
Vab  1.05sin(5000t  93.720 )
The resultant voltage across capacitor (In steady state).
Vab  5  1.05sin(5000t  93.720 )
The average value Vab ( avg )  5 V
Hence, the correct answer is 5.
1 if a  t  b
Q.10 The bilateral Laplace transform of a function f (t )   is
0 otherwise
a b e z ( a  b)
(A) (B)
s s
e as  ebs e  ( a b )
(C) (D)
s s
Ans. (C)
1 if a  t  b
Sol. Given : f (t )  
0 otherwise

f (t ) can be written as
f (t )  u (t  a )  u (t  b)
Taking Laplace transform on both sides, we get
e as  ebs
F ( s) 
s
PAGE
11

Hence, the correct option is (C).


Q.11 A mod-n counter using a synchronous binary up-counter with synchronous clear input is shown in the
figure. The value of n is _______.

Ans. (7)
Sol. Given counter is shown below

CLEAR input is active low, means if output of the NAND gat is “0” then counter would be clear.
Hence QB QC  11
and QA QD  00
Counter is cleared after
QAQB QC QD  0110
Hence MOD of the counter is 7 as it counts states from o to 6.

Q.12 The electric field of a uniform plane electromagnetic wave is E  (aˆ x  j 2aˆ y ) exp[ j (210 t  0.2 z )] .
7

The polarization of the wave is


(A) Right handed circular (B) Right handed elliptical
(C) Left handed circular (D) Left handed elliptical
Ans. (D)
Sol. Given : E  (ax  j 2a y ) exp  j (2107 t  0.2 z) 

Ex  e j (2 10 t 0.2 z )
7
 Ex 0  1

Ey  2e j (210 t 0.2 z )
7
 Ey0  2
PAGE
12

E y leads E x by an angle of 90 0 , E x 0  E y 0 and wave is travelling in +z direction

 The given uniform plane wave is left hand elliptically polarized.


Hence option (D) is correct

 
Q.13 The signal cos 10t   is ideally sampled at a sampling frequency of 15 Hz. The sampled signal is
 4
 sin( t )   
passed through a filter with impulse response   cos  40t   . The filter output is
 t   2
15   15  sin( t )   
(A) cos  40t   (B)   cos 10t  
2  4 2  t   4
15   15  sin( t )   
(C) cos  10t   (D)   cos 10t  
2  4 2  t   2
Ans. (A)
 
Sol. Given : x(t )  cos 10 t  
 4
f s  15Hz

Neglect the phase shift , as it can be inserted in the final result. So
4
x1 (t )  cos(10t )
Fourier transform of cos(2f 0t ) is
1
cos 2f 0t 
F .T .
 [( f  f 0 )  ( f  f 0 )]
2
Fourier transform of x1 (t ) can be written as,
1
X 1 ( f )  [( f  5)  ( f  5)]
2
Impulse response
 sin t   
h(t )    cos  40 t  
 t   2
PAGE
13

h(t )  sin c(t )sin(40t ) ….. (i)


Fourier transform of rectangular signal is given by,
 t  F .T .
A rect     A sinc( f )

Using duality property,
 f 
A sinc  t   
F .T .
 A rect  

sinc  t  
F .T .
 rect  f 
Fourier transform of sin(2f 0t ) is given by,
1
sin(2f 0t ) 
F .T .
 [( f  f 0 )  ( f  f 0 )]
2j
Taking Fourier transform of equation (i), we get
1
H ( f )  rect ( f )  [( f  20)  ( f  20)]
2j
1
H ( f )  [rect ( f  20)  rect ( f  20)]
2j
Now, X 1 ( f ) repeats with a f s  15kHz. The expression of sampled signal is given by,

X 1s ( f )  f s  X ( f  nf )
n 
1 s

The spectrum of sampled signal is shown in figure.

The spectrum of H ( f ) is shown in figure.

So when this sampled signal is passed through the filter having response H ( f ), we get the signal which
will be in the range of 19.5 to 20.5 Hz. Hence, we get
15 1
X 1s ( f ) H ( f )   [( f  20)  ( f  20)]
2 2j
PAGE
14

15
X 1s ( f ) H ( f ) 
[( f  20)  ( f  20)]
4j
Taking inverse Fourier transform, we get
15
xr (t )  sin(40t )
2
15  
xr (t )   cos 40t  
2 2

This is recovered signal. Hence, applying phase shift , we get
4
15   
xr (t )  cos  40t   
2  2 4
15  
xr (t ) cos  40t  
2  4
Hence, the correct option is (A).
Q.14 A sinusoidal signal of amplitude A is quantized by a uniform quantizer. Assume that the signal utilizes
all the representation levels of the quantizer. If the signal to quantization noise ratio is 31.8 dB, the
number of levels in the quantizer is ______.
Ans. 32
Sol. Given : SQNR = 31.8dB
For sinusoidal signal, signal to quantization noise ratio is given by,
( SQNR)dB  1.76  6n
31.8  1.76  6 n
6 n  30
n5
The relationship between number of quantization level and number of bits is given by,
L  2n
L  25  32
Hence, the correct answer is 32.
Q.15 Let the signal f (t )  0 outside the interval [T1 , T2 ] , where T1 and T2 are finite. Furthermore, f (t )   .
The region of convergence (ROC) of the signal’s bilateral Laplace transform F(s) is
(A) A parallel strip containing the j axis
(B) A parallel strip not containing the j axis
(C) The entire s-plane
(D) A half plane containing the j axis
Ans. (C)
Sol. Given : Signal f (t )  0 outside the interval T1 , T2  where T1 and T2 are finite.
Therefore signal is a finite duration signal.
PAGE
15

For a finite duration signal, ROC is entire s-plane.


Hence, the correct option is (C).
Q.16 The 2-port admittance matrix of the circuit shown is given by

 0.3 0.2  15 5 


(A)   (B)  
 0.2 0.3  5 15
3.33 5   0.3 0.4 
(C)   (D)  
 5 3.33  0.4 0.3
Ans. (A)
Sol. Given circuit is shown below.

 ya  yb  yb 
For above circuit  y   yb  yc 
  yb
1 1 1
In question ya  , yb  , yc 
10 5 10
1 1 1   3 1
10  5     
 y   5

10 5

 1 1 1  1
 
3 
 5 5 10   5 10 
 0.3  0.2 
 y   
  0.2 0.3 
Hence, the correct option is (A).
az  b
Q.17 Let f ( z )  . If f ( z1 )  f ( z2 ) for all z1  z2 , a  2, b  4 and c  5 , then d should be equal to
cz  d
_____.
Ans. 10
az  b
Sol. Given : f ( z ) 
cz  d
If f ( z1 )  f ( z2 ) for all z1  z2
where a  2, b  4 and c  5
PAGE
16

As f ( z1 )  f ( z2 )
az1  b az2  b

cz1  d cz2  d
(2 z1  4) (5 z2  d )  (2 z2  4) (5 z1  d )
10 z1 z2  2dz1  20 z2  4d  10 z1 z2  20 z1  2dz2  4d
2d ( z1  z2 )  20( z1  z2 )
 d  10 Ans.
Hence, the correct answer is 10.
Q.18 The voltage (Vc ) across the capacitor (in Volts) in the network shown in ______.

Ans. 100
Sol. For series RLC circuit

The voltage across RLC are


I
VR  IR, VL  jLI and VC 
jC
Supply voltage VS is given by,
VS  VR  VL  VC
VR  80 V, VL  40 V and VS  100 V
If VL  VC ,

If VC  VL ,
PAGE
17

The resultant supply voltage


VS  VR2  (VL  VC ) 2

100  (80)2  (40  VC ) 2


(40  VC )  60
VC  100 V,  20 V
VC   ve hence VC  100 V
Hence, the correct answer is 100.
Q.19 If the circuit shown has to function as a clamping circuit then which one of the following conditions
should be satisfied for the sinusoidal signal of period T ?

(A) RC  T (B) RC  0.35 T


(C) RC  T (D) RC  T
Ans. (D)
Sol. For an ideal clamping circuit once the capacitor is charged it should not discharge.
Hence discharging time constant (RC) must be much larger than the time period of input signal.
i.e. RC  T
Hence, the correct option is (D).
Q.20 In the bistable circuit shown, the ideal op-amp has saturation levels of  5 V . The value of R1 (in k )
that gives a hysteresis width of 500 mV is _____.

Ans. 1
Sol. Given circuit is shown below.
PAGE
18

Voltage v  is given by,


V R V R
v   in 2  out 1
R1  R2 R1  R2
When Vout   VCC ,
V R V R
Then v   V1  in 2  CC 1
R1  R2 R1  R2
Again Vout  VCC if v   0
Vin R2 V R
i.e.  CC 1  0
R1  R2 R1  R2
R
Vin  VCC 1 …… (i)
R2
When Vout   VCC
Vin R2 V R
Then v   CC 1
R1  R2 R1  R2
Again Vout   VCC if v   0
Vin R2 V R
 CC 1  0
R1  R2 R1  R2
VCC R1
Vin  …… (ii)
R2
Transfer characteristics is shown below.

VCC R1 V R
Where V1  and V2  CC 1
R2 R2
Hysteresis is given by,
PAGE
19

VCC R1  VCC R1 
VH  V1  V2   
R2  R2 
R1
VH  2VCC
R2
Given VH  0.5 V
R1
2 5  0.5
20
R1  1 k
Hence, the correct answer is 1.
Q.21 In the figure shown, the output Y is required to be Y  AB  C D . The gates G1 and G2 must be,

(A) NOR, OR (B) OR, NAND


(C) NAND, OR (D) AND, NAND
Ans. (A)
Sol. Given logic circuit is shown below

From above figure


N  C  D  C.D
If G1 is NOR gate

M  A  B  A.B
If G2 is OR gate

Y M N
Y  A.B  C .D
Therefore G1 is NOR gate and G2 is OR gate
Hence, the correct option is (A).
PAGE
20

Q.22 An n-type silicon sample is uniformly illuminated with light which generates 10 20 electron-hole pairs per
cm 3 second. The minority carrier lifetime in the sample is 1s . In the steady state, the hole
concentration in the sample is approximately 10 x , where x is an integer. The value of x is ________.
Ans. 14
Sol. Given :
Electron-hole pairs generation rate
 1020 /cm3 /sec
Minority life time (i.e. hole),
 p  1 m  106 sec
Number of hole generated in
1 sec  1020 /cm3
6
Number of holes left after 10 sec
 1020 10 6  1014 /cm3
Hence x = 14
Hence, the correct answer is 14.
Q.23 The value of x for which all the Eigen-values of the matrix given below are real is
10 5  j 4 
x 20 2 

 4 2 10 
(A) 5  j (B) 5  j
(C) 1  5 j (D) 1  5 j
Ans. (B)
10 5  j 4 
Sol. 
Given : Let the matrix A   x 20 2 
 4 2 10 
Transpose conjugate of A is given by,
10 5  j
T
4   10 x 4 
 
A  ( A)   x
T
20 2   5  j 20 2 
 
 4 2 10   4 2 10 
As Eigen values of A are real it means A is “Hermitian matrix” i.e. A  A
 10 x 4  10 5  j 4 
5  j 20 2    x 20 2 
  
 4 2 10   4 2 10 
x  5  j and x  5  j
 x  5 j
Hence, the correct option is (B).
PAGE
21

Q.24 In an 8085 microprocessor, which one of the following instructions changes the content of the
accumulator?
(A) MOV B, M (B) PCHL
(C) RNZ (D) SBI BEH
Ans. (D)
Sol. Content of accumulator are changed by the arithmetic and logical operations SBI BEH is a arithmetic
operation.
Hence, the correct option is (D).
Q.25 By performing cascading and/or summing/differencing operations using transfer function blocks G1 ( s )
and G2 ( s) , one CANNOT realize a transfer function of the form
G1 ( s )
(A) G1 ( s) G2 ( s) (B)
G2 ( s )
 1   1 
(C) G1 ( s )   G2 ( s )  (D) G1 ( s)   G2 ( s ) 
 G1 ( s )   G1 ( s) 
Ans. (B)
Sol. Since we have only G1 ( s ) and G2 ( s).
For option (A),

1 
For option (C), G1   G2   1  G1G2
 G1 

1 
For option (D), G1   G2   1  G1G2
 G1 

Hence, the correct option is (B).


Q.26 to Q.55 carry two marks each
Q.26 A function of Boolean variables, X, Y and Z is expressed in terms of the minterms as
F ( X , Y , Z )  (1, 2, 5, 6, 7)
Which one of the product of sums given below is equal to the function F ( X , Y , Z ) ?
(A) ( X  Y  Z )  ( X  Y  Z )  ( X  Y  Z )
PAGE
22

(B) ( X  Y  Z )  ( X  Y  Z )  ( X  Y  Z )
(C) ( X  Y  Z )  ( X  Y  Z )  ( X  Y  Z )  ( X  Y  Z )  ( X  Y  Z )
(D) ( X  Y  Z )  ( X  Y  Z )  ( X  Y  Z )  ( X  Y  Z )  ( X  Y  Z )
Ans. (B)
Sol. . Method 1 .
Given : F ( X , Y , Z )  (1, 2,5, 6, 7)
K-map for SOP form is shown below

K-map for POS form is shown below

Hence F ( XYZ )  ( X  Y  Z )( X  Y  Z ) ( X  Y  Z )
Hence, the correct option is (B).
. Method 2 .
F ( X , Y , Z )  (1, 2,5, 6, 7)
Expression is given in SOP form but option are given in POS form Hence, Convert it into POS form
F ( XYZ )  m(1, 2,5, 6, 7)  m(0,3, 4)
F ( X , Y , Z )  ( X  Y  Z )( X  Y  Z ) ( X  Y  Z )
Hence, the correct option is (B).
Q.27 An air-filled rectangular waveguide of internal dimensions a cm  b cm (a  b) has a cutoff frequency of
6 GHz for the dominant TE10 mode. For the same waveguide, if the cutoff the TM 11 mode is 15 GHz, the
cutoff frequency of the TE01 mode in GHz is ________.
Ans. 13.75
Sol. Given : For TE10 mode, f c  6 GHz
c
fc  (for TE10 mode)
2a
3 108
6 10  9

2a
a  2.5 cm
For TM 11 mode, f c  15 GHz
PAGE
23

2 2
c m n
fc     
2  a  b
Putting m = n = 1, we get
c 1 1
fc  
2 a 2 b2
3 108 1 1
15 109   2
2 6.25 b
2.5
b
5.25
f c for TE01 is given by,

c 3 1010
fc    5.25
2b 2  2.5
f c  13.75 GHz
Hence, the correct answer is 13.75.

 X n n   is an independent and identically distributed (i.d.) random process


n
Q.28 X n equally likely to be 1

or –1. Yn n   is another random process obtained as Yn  X n  0.5 X n 1 . The autocorrelation function
n

Yn n  
n
of denoted by R y [ k ] , is

(A) (B)

(C) (D)

Ans. (B)
Sol. The autocorrelation function is defined as
RY (k )  Ry (n, n  k )
RY (k )  E Y (n).Y (n  k )
Now, we have
Y (n)  x(n)  0.5 x(n  1)
PAGE
24

So,  E ( x(n)  0.5x(n  1))( x(n  k )  0.5x(n  k  1)) 


 E[( x(n).x(n  k )  x(n)0.5 x(n  k  1)  0.5 x(n  1).x( n  k )  0.25 x( n  1)  x( n  k  1)]
 E[( x(n).x(n  k )  0.5 E[ x(n)  x(n  k  1)]  0.5 E[ x(n  1) x(n  k ))]
 0.25 E[ x(n  1) x(n  k  1)]
 Rx (k )  0.5Rx (k  1)  0.5Rx (k  1)  0.25Rx (k )
Ry (k )  1.25Rx (k )  0.5Rx (k 1)  0.5Rx (k 1)
Rx (k )  E[ x(n).x(n  k )]
For k  0 , we obtain
Rx (0)  E[ x2 (n)]
1 1
 12.  ( 1) 2 
2 2
1
Again, for k  0 , we have
Rx (k )  E[ x(n).x(n  k )]
=0 Since E[ x(n)]  0, E[ x(n  k )]  0
Hence, we get
Ry (0)  1.25Rx (0)  0.5Rx (1)  0.5Rx (1)
= 1.25
Ry (1)  1.25Rx (1)  0.5Rx (0)  0.5Rx (2)
= 0.5
Ry (1)  1.25Rx (1)  0.5Rx (2)  0.5Rx (0)
= 0.5
Ry ( k ) for k other than 0, 1 and -1 = 0. Thus, the autocorrelation function Ry ( k ) is plotted as

Hence, the correct option is (B).


Q.29 In MOS capacitor with an oxide layer thickness of 10 mm. The maximum depletion layer thickness is
100 mm. The permittivities of the semiconductors and the oxide layer are  s and ox respectively.
Assuming  s ox  3 the ratio of the maximum capacitance to the minimum capacitance of this MOS
capacitor is ____.
Ans. 4.33
Sol.
PAGE
25

This is equivalent to two capacitance are connected in series.

Capacitance-voltage curve at high frequency is shown below.

 ox
CG (max)  Cox 
tox
But CG (min) will be minimum when C dp is minimum because Cox and C dp is in series and C dp will be
minimum when depletion width is maximum.
s
Cdp (min) 
wd (max)

CG (max) Cox Cox  Cdp (min)


 
CG (min) Cox  Cdp (min) Cdp (min)
Cox  Cdp (min)
Cox  w (max)
 1  1  ox  d
Cdp (min) tox s

   w (max)  1 100
 1   ox  d   1   4.33
  s  tox  3 10
Hence, the correct answer is 4.33.
Q.30 An LC tank circuit consists of an ideal capacitors C connected in parallel with a coil of inductance L
having an internal resistance R. The resonant frequency of the tank circuit is
PAGE
26

1 1 C
(A) (B) 1  R2
2 LC 2 LC L
1 L 1  2 C 
(C) 1 2 (D) 1  R 
2 LC RC 2 LC  L
Ans. (B)
Sol. For given tank circuit

1 1
Impedance   jC
Z R  jL
1
Admittance Y   jC
R  jL
( R  jL)
Y  jC
R 2  (L)2
R  L 
Y  j  C  2 2
R  ( L )
2 2
 R  ( L ) 
At resonance imaginary part of Y should be zero. Hence
L
C 2 0
R  (L) 2
L
R 2  ( L ) 2 
C
L
2 L2   R2
C
2
1 R
2   
LC  L 
Resonance frequency
2
1 R
0   
LC  L 

1 1 R2
f0  
2 LC L2
1 CR 2
f0  1
2 LC L
PAGE
27

Hence, the correct option is (B)


Q.31 Let the random variable X represent the number of times a fair coin needs to be tossed till two
consecutive heads appear for the first time. The expectation of X is ________.
Ans. 1.5
Sol. Let X be random variable which denote number of tosses to get two heads.
1 1
P( X  2)  HH  
2 2
1 1 1
P( X  3)  THH   
2 2 2
4
1
P( X  4)  TTHH   
2
So, E ( X )   XP( X )
2 3 4
1 1 1
E ( X )  (2)    3    4   .........
2 2 2
2 3 4
1 1 1
Again, let s  (2)    3    4   ......... …(i)
2 2 2
3 4 5
s 1 1 1
 (2)    3    4   ......... …(ii)
2 2 2 2
Subtracting equation (ii) and (i),
2 3 4
s 1 1 1
 (2)      [3  2]    [4  3].........
2 2 2 2
2 3 4 5
s 1 1 1 1
 (2)            .........
2 2 2 2 2
2 4 5
s 1 1 1 1
          .........
2 2 2 2 2
3
1
s 1  2   1 
2

   
2 2 1 1  2 
2
s 1 1
 
2 2 4
1
s  1
2
3
s
2
PAGE
28

s  1.5
3
Hence, E( X ) 
 1.5
2
Hence, the correct answer is 1.5.
Q.32 The energy band diagram and the electron density profile n(x) in a semiconductor are shown in the
 qx 
 
figures. Assume that n( x)  1015 e kT cm-3 , with   0.1 V/cm and x expressed in cm. Given
kT D kT
 0.026 V, Dn  36 cm 2s -1 and  . The electron current density (in A/cm 2 ) at x  0 is
q  q

(A)  4.4 102 (B) 2.2 102


(C) 0 (D) 2.2  10 2
Ans. (C)
Sol. The concentration of doping is not uniform through the semiconductor, thus to maintain equilibrium
internal electric field is generated due to which the band of the semiconductor is varying with slope –
0.1 eV/cm.
The generated electric field opposes the diffusion of carries due to concentration gradient and thus there
will be no current flows inside the semiconductor.
Hence, the correct option is (C).
1
Q.33 The transfer function of a mass-spring-damper system is given by G ( s )  . The frequency
Ms  Bs  K
2

response data for the system are given in the following table.
 in rad/s G( j) in dB arg G( j) in deg
0.01 – 18.5 – 0.2
0.1 – 18.5 – 1.3
0.2 – 18.4 – 2.6
1 – 16 – 16.9
2 – 11.4 – 89.4
3 – 21.5 – 151
5 – 32.8 – 167
10 – 45.3 – 174.5
The unit step response of the system approaches a steady state value of _______.
Ans. 0.12
PAGE
29

1
Sol. Given : G ( s) 
Ms  Bs  K 2

System is shown below,

Output Y ( s ) is given by,


Y ( s )  G ( s )U ( s )
1 1
Y ( s)  
( Ms  Bs  K ) s
2

Steady state value is given by,


1
y ()  lim sY ( s)  lim
s 0 s 0 ( Ms  Bs  K )
2

1
y ( ) 
K
Now, at   0.01 rad/sec, G( j) dB  18.5

20log G( j)  18.5


1
20 log  18.5
K
Alternatively :
From Bode plot concept :
Initial value of G ( j) is constant.
20log dc gain  18.5
1
20 log  18.5
K
1  18.5
log  
K  20
18.5
1
y ()   10 20  0.1188
K
y ()  0.12
Hence, the correct answer is 0.12.
Q.34 The figure shows a binary counter with synchronous clear input. With the decoding logic shown, the
counter works as a
PAGE
30

(A) mod-2 counter (B) mod-4 counter


(C) mod-5 counter (D) mod-6 counter
Ans. (C)
Sol. Given binary counter is shown below

CLR input is active low. Hence if output of EX-NOR gate is “0” then counter would be clear.
Hence Q3Q2  01
and Q1Q0  00
Required input state Q3Q2Q1Q0  0100
So number of states counted are 0 to 4.
Hence it is a MOD 5 counter
Q.35 The state variable representation of a system is given as
0 1  1 
x  x, x(0)   
0 1 0
y  [0 1]x
The response y (t) is
(A) sin(t ) (B) 1  et
(C) 1  cos(t ) (D) 0
Ans. (D)
0 1  1 
Sol. Given : x  x, x(0)    and y  [0 1]x …. (i)
0 1 0
State equation is given by,
x  Ax  Bu …. (ii)
Output state equation is given by,
y  Cx  Du …. (iii)
On comparing equation (i), (ii) and (iii), we get
PAGE
31

0 1 
A  , C   0 1
0 1
 s 0  0 1   s 1 
[ sI  A]     
0 s  0 1 0 s  1
Adj sI  A
 sI  A 
1

sI  A
 s  1 1
Adj[ sI  A]  
 0 s 
sI  A  s( s  1)
 s 1 1  1 1 
 s ( s  1) s ( s  1)   s s ( s  1) 
[ sI  A]1    
 s   1 
 0 s( s  1)  
0
( s  1) 

Taking inverse Laplace transform, we get
1 1  e  t 
(t )  L1[ sI  A]1   
0 et 
If input is not given then we consider zero input response.
x(t) = ZIR
x(t )  L1[ sI  A]1 x(0)
1 1  e  t  1  1 
x(t )       
0 et  0 0
y (t )  Cx(t )
1 
y (t )   0 1    0
0 
Hence, the correct option is (D).
Q.36 In the ac equivalent circuit shown, the two BJTs are biased in active region and have identical
parameters with   1 . The open circuit small signal voltage gain is approximately _____.
PAGE
32

Ans. –1
Sol. Transconductance of a transistor is given by
I
gm  C
VT
Since both the transistors are biased at the same collector current, hence transconductance will be same
for both the transistor.
The small signal equivalent is shown below.

g m 1  g m2
1
Therefore, Vout   g m1Vi 
g m2
Vout  Vin
Vout
Gain AV   1
Vin
Hence, the correct answer is –1.
Q.37 For the voltage regulator circuit shown, the input voltage (Vin ) is 20 V  20% and the regulated output
voltage (Vout ) is 10 V. Assume the opamp to be ideal. For a load RL drawing 200 mA, the maximum
power dissipation in Q1 (in Watts) is ______.

Ans. (2.8)
Q.38 A dc voltage of 10 V is applied across an n -type silicon bar having a rectangular cross-section and
length of 1 cm as shown in figure. The donor doping concentration N D and the mobility of electrons  n
are 1016 cm – 3 and 1000 cm  2 V 1s 1 , respectively. The average time (in μs ) taken by the electrons to
move one end of the bar to other end is ________.
PAGE
33

Ans. 100
Sol. Electric field is given by,
V
E
d
10 V
E  10 V/cm
1 cm
Drift velocity is given by,
Vd  E  1000 10
Vd  104 cm/sec
The average time (in s ) taken by the electrons to move from one end of the bar to other end is given
by,
L 1
t  4
V 10
t  10 4 sec  100 sec
Hence, the correct answer is 100.
Q.39 Consider two real sequences with time-origin marked by the bold value,
x1[n]  {1, 2, 3, 0}, x2 [n]  {1, 3, 2, 1}
Let X 1 (k ) and X 2 (k ) be 4-point DFTs of x1[n] and x2 [n] , respectively.
Another sequence x3 [ n] is derived by taking 4-point inverse DFT of X 3 (n)  X 1 (k ) X 2 (k ) .
The value of x3 [ n] is ________.
Ans. 11
Sol. Given : x1  n  1, 2, 3, 0
And x2  n  1, 3, 2, 1
X 3 (k )  X 1 (k )  X 2 (k )
Therefore, x3  n  x1  n  x2  n
Where  denotes the circular convolution
1 0 3 2  1 
2 1 0 3 3
x3  n    
3 2 1 0 2
  
 0 3 2 1  1 
PAGE
34

1  0  6  2   9 
 2  3  0  3  8 
x3  n     
3  6  2  0  11
   
 0  9  4  1 14 
x3  n  9 8 11 14
Therefore, x3  2  11
Hence, the correct answer is 11.
Q.40 Input x (t ) and output y (t ) of an LTI system are related by the differential equation
y "(t )  y '(t )  6 y (t )  x(t ) . If the system is neither causal nor stable, the impulse response h (t ) of the
system is
1 1 1 1
(A) e3t u (t )  e 2t u (t ) (B)  e3t u (t )  e 2t u (t )
5 5 5 5
1 3t 1 1 1
(C) e u (t )  e 2t u (t ) (D)  e3t u (t )  e 2t u (t )
5 5 5 5
Ans. (B)
Sol. Given : y "(t )  y '(t )  6 y (t )  x(t )
Taking Laplace transform on both sides, we get
s 2Y ( s)  sY ( s)  6Y ( s)  X ( s)
Y (s)  s 2  s  6  X (s)
Transfer function H ( s ) is given by
Y (s) 1 1
H (s)   2 
X ( s ) ( s  s  6) ( s  3) ( s  2)
1  1
   
H ( s)   5

5
( s  3) ( s  2)
Taking inverse Laplace transform on both sides, considering system is neither causal nor stable.
1 1
h(t )   e3t u (t )  e 2t u (t )
5 5
Hence, the correct option is (C).
 sin(4t )
Q.41 The value of the integral 

12 cos(2t )
4t
dt is _______.

Ans. 3

sin(4t )
Sol. Given : I  12 cos(2t )

4t
dt
PAGE
35

 
12 cos 2t sin 4t 3 2cos 2t sin 4t
I  2 dt   dt
4 0 t 0 t
3  sin 6t sin 2t 
 
I   dt   dt   2cos A sin B  sin ( A  B)  sin ( A  B)
 0 t 0
t 
Taking Laplace transform of both sides, we get
3   sin 6t   sin 2t 
I  L   L   with s  0
  t   t 
3 
 
6 2
I   2 ds   2 ds  with s  0
  s s  36 2
s
s  4 2


3 1  s  1  s 
I  6  tan 1    2  tan 1    with s  0
 6  6  2  2   s
3  1  s   s 
I  tan ()  tan 1    tan 1 ()  tan 1    with s  0
  6   2  
3   
I   tan 1 0   tan 1 0 
 2 2 
3
I     3 Ans.

Hence, the correct answer is 3.
Q.42 Let x(t )  a s (t )  s (t ) with s(t )  e4t u (t ) , where u (t ) is unit step function. If the bilateral Laplace
transform of x (t ) is
16
X ( s)   4  Re{s}  4;
s  162

Then the value of  is _______.


Ans. –2
Sol. Given : x(t )  s (t )  s (t )
and s(t )  e 4t u (t )
Hence, x(t )   e 4t u (t )   e4t u ( t )
Taking Laplace transform on both sides, we get
 ()
X (s)  
( s  4) ( s  4)
( s  4)  ( s  4)
X (s) 
s 2  16
(  ) s  ( 4  4)
X ( s) 
s 2  16
16
Comparing with X ( s) 
( s  16)
2
PAGE
36

We get (  )  0
(  1)  0
Since   0 therefore   1.
And  4  4  16
 4(  1)  16
 4(1  1)  16
  2
Hence, the correct answer is –2.
Q.43 The electric field of a plane wave propagating in a lossless non-magnetic medium is given by following
expression

E ( z , t )  ax 5cos(2109 t  z )  a y 3cos(2109 t   z  )
2
(A) Right Hand Circular (B) Left Hand Elliptical
(C) Right Hand Elliptical (D) Linear
Ans. (B)
Sol. Given : Ex  5cos(2109 t z)
 
E y  3cos  2 109 t   z  
 2
E x 0  5, E y 0  3

E x leads E y by an angle of 90 0 , Ex 0  E y 0 and wave is travelling in  z direction.


 The wave is left elliptical polarized plane-wave.
Hence, the correct option is (B).
Q.44 Let X  {0, 1} and Y  {0, 1} be two independent binary random variables. If P( X  0)  p and
P (Y  0)  q , then P ( X  Y  1) is equal to
(A) pq (1  p )(1  q ) (B) pq
(C) p(1  q) (D) 1  pq
Ans. (D)
Sol. Given : P ( x  0)  p
PAGE
37

So, P( x  1)  1  p
P( y  1)  q
and P( y  1)  1  q
Let, Z  X Y
X Y Z
0 0 0
0 1 1
1 0 1
1 1 2
So, P( z  1)  1  P{x  0 and y  1}  P{x  1and y  0}  P{x  1and y  1}
 1  P{x  0 and y  0}
 1  Pq
Hence, the correct option is (D).
1
Q.45 If C denotes the counterclockwise unit circle, the value of the contour integral
2j  Re{z}dz is _____.
c

Ans. 0.5
Sol. Given : C denotes the counterclockwise unit circle,
Re( z )  x as z  x  iy
As curve C indicates unit circle in anticlockwise orientation i.e. z  1
z  1 ei and   0 to 2

As z  ei
dz  iei and x  1 cos 
2
1 1
Hence, I
2i C
 x dz  
2i 0
1 cos  iei d 
2 2
1 1
I 
2 0
cos  ei d  
2 0
cos (cos   i sin ) d 

1  
2 2
I      sin  cos  d  
2
cos d i
2  0 0 
PAGE
38

1 1 
2 2
I    (1  cos 2) d   i  sin 2 d  
2 2  0 0 
1  2 
2 2
I 0

4  0
cos 2 d   i 0 sin 2 d 
1 1
I (2  0)  0  0   0.5 Ans.
4 2
Hence, the correct answer is 05.
Q.46 Consider a binary, digital communication system which uses pulses g (t ) and – g (t ) for transmitting bits
over an AWGN channel. If the receiver uses a matched filter, which one of the following pulses will
give the minimum probability of bit error?
(A) (B)

(C) (D)

Ans. (A)
Sol. Let N (t ) be the noise at the output of filter.
Variation of N (t )  E  N 2 (t )   E  N (t )
Since the input noise is zero mean.
Output noise mean is also zero.
 
E  N (t )  E (W (t ))    h(t ) dt 
  
 
E W (t )  0
W (t ) is white noise, so
var  N (t )  E  N 2 (t )   RN (0)
Since RN ()  h()* h( )* Rw ()
N0
and RN ()   ()
2
N0
RN ()   h() * h(  )  
2
PAGE
39


N0
RN () 
2  h(k )  h(  k ) dk



N N0
RN (0)  0  h (k ) dk 
2
(3 A2 )
2 
2
3 2
RN (0)   A  N0
2
Hence, the correct option is (A).
Q.47 In the circuit shown, the initial voltages across the capacitors C1 and C2 and 1 V and 3 V, respectively.
The switch is closed at time t  0 . The total energy dissipated (in Joules) in the resistor R until steady
state is reached, is _______.

Ans. 1.5
Sol. . Method 1
Given : V1 (0 )  1 V
V2 (0  )  3 V
For t  0 , change the circuit in Laplace domain

3 1 2

I (s)  s s  s
1 1 30 s  1  3
10  
3s s 3s
6 6
I ( s)  
30 s  4  4 
30  s  
 30 
0.2
I (s) 
 2
s 
 15 
Taking inverse Laplace transform, we get
PAGE
40

2
 t
i (t )  0.2e 15

Power dissipated in resistor is given by,


p(t )  i 2 (t ) R
4t 4t
 
p (t )  (0.2) 2 e 15
10  0.4 e 15

Total energy dissipated from t = 0 to t   is given by,


  4t

E   p dt  0.4 e 15
dt
0 0


 4t 
 e 15 
E  0.4  
  4  
  15  
0

 0.4 15
E 0  1  1.5 Joules
4
Hence, the correct answer is 1.5.
. Method 2 .
1
Initial energy  (C1V12  C2V22 )
2
1
(3 12  1 32 )  6 J
2
Final energy stored in capacitor
1

(C1  C2 )V 2
2
As we know total change before and after will be same
Q(0 )  Q(0 )
i.e. C1V1  C2V2  (C1  C2 )V
1 3  1 3  (1  3)V
V  1.5 V
1
 Final energy  (1  3)  (1.5) 2  4.5 J
2
Energy dissipated = (6  4.5) J  1.5 J
Hence, the correct answer is 1.5.
Q.48 The output of a standard second-order system for a unit step input is given as
2 t  
y (t )  1  e cos  3t   . The transfer function of the system is
3  6
PAGE
41

2 1
(A) (B)
( s  2)( s  3) s  2s  1
2

3 4
(C) (D)
s  2s  3
2
s  2s  4
2

Ans. (D)
2 t  
Sol. Given : y (t )  1  e cos  3t   …… (i)
3  6
Step response of second order system is given by,
e nt
c(t )  1  sin(d t  ) …… (ii)
1  2

 1  21 1

where d  n 1   ,   cos   tan 
2

  
On comparing equation (i) and (ii), we get
1 2
 n  1 and 
1  2
3
1

2
n  2
Alternatively,
From equation (i),
2 t   
y (t )  1  e sin  3t    …… (iii)
3  6 2
On comparing equation (i) and (iii), we get

  cos 1  
3
  0.5
Hence, the correct option is (D).
Q.49 Two half-wave dipole antennas placed as shown in the figure are excited with sinusoidally varying
currents of frequency 3 MHz and phase shift of  2 between them (the element at the origin leads in
phase). If the maximum radiated E-field at the point P in the x-y plane occurs at an azimuthal angle of
600, the distance d (in meters) between the antennas is __________.
PAGE
42

Ans. 50
Sol. Two half-wave dipole antennas are shown below.

Array factor is given by,


AF  d cos   
Where   900
Phase constant is given by,
2 2 2
  
 3 10 8
100
3 10 6

Since maximum ( AF  0) is at   600


2
0 d cos 600  900
100
 2 1
 d
2 100 2
d  50 m
Hence, the correct answer is 50.
Q.50 The diode in the circuit given below has VON  0.7 V but is ideal otherwise. The current (in mA) in the
4 kΩ resistor is __________.
PAGE
43

Ans. 0.6
Sol. Given bridge circuit is in balance condition.
As product of opposite arms are equal
i.e. (2 k)  (6 k)  (4 k)  (3k)
12 k  12 k
Therefore current will not flow through the diode

Current I is given by
9 9
I 1 
96 15
I  0.6 A
Hence, the correct answer is 0.6.
Q.51 In the circuit shown, the Norton equivalent resistance (in Ω ) across terminals a-b is _______.

Ans. 1.33
Sol. . Method 1 :
To find the Norton equivalent resistance, apply a voltage source of Vdc and current I dc flows to the
network.
PAGE
44

Apply nodal analysis at node a,


Vdc  4 I Vdc Vdc
   I dc
2 2 4
Vdc
Where I 
4
Hence, 4 I  Vdc
Vdc  Vdc Vdc Vdc
   I dc
2 2 4
3Vdc
 I dc
4
Vdc 4
 Rth    1.33 
I dc 3
Hence, the correct answer is 1.33.
. Method 2 :
Voltage across 4  resistor = 4I

Applying KVL in the loop shown in figure.


 4 I  2i  4 I  0
i 0

4 2 4
Req   4 || 2    
42 3
Hence, the correct answer is 1.33.
N0
Q.52 A zero mean white Gaussian nose having power spectral density is passed through an LTI filter
2
whose impulse response h (t ) is shown in the figure. The variance of the filtered noise at t  4 is
PAGE
45

3 2 3 2 1 2
(A) A N0 (B) A N0 (C) A2 N0 (D) A N0
2 4 2
Ans. (A)
Sol. Let N (t ) be the noise at the output of filter.
Variation of N (t )  E[ N 2 (t )]  {E[ N (t )]}2
Since the input noise is zero mean.
Output noise mean is also zero.
 
E[ N (t )]  E (W (t ))    h(t ) dt 
  
W (t ) is white noise, so
var ( N (t ))  E[ N 2 (t )]  RN (0)
Since RN ()  h()* h()* R ()
N0
and RN ()   (t )
2
N0
RN ()  [h() * h( )] 
2

N
RN ()  0   h(k )  h(  k )dk
2 

N N0
RN (0)  0  h (k ) dk 
2
(3 A2 )
2 
2
3 2

 A  N0
2
Hence, the correct option is (A).
Q.53 Assuming that the op-amp in the circuit shown below is ideal, the output voltage V0 (in volts) is ____.
PAGE
46

Ans. 12
Sol. V  1V and V  0 V
V  V
So, V0  Vsat  12 V
Hence, the correct answer is 12.
dx
Q.54 Consider the differential equation  10  0.2 x with initial conduction x (0)  1 . The response x (t ) for
dt
t  0 is
(A) 2  e0.2t (B) 2  e 0.2t
(C) 50  49e 0.2t (D) 50  49e0.2t
Ans. (C)
Sol. . Method 1 .
Given : The differential equation
dx
 10  0.2 x, x(0)  1
dt
dx
The equation  0.2 x  10
dt
Taking Laplace Transform,
 dx 
L    0.2 L  x   10 L 1
 dt 
10
 sX (s)  x(0)  0.2 X ( s) 
s
10
( s  0.2) X ( s )  1 
s
1 10 1 10  1 1 
X (s)      [By applying partial fraction]
s  0.2 s ( s  0.2) s  0.2 0.2  s s  0.2 

50 49
X ( s)  
s s  0.2
Taking Laplace inverse,
x(t )  50  49e0.2t
So, the correct option is (C).
PAGE
47

. Method 2 .
d
Let D
dt
The equation is
( D  0.2) x  10
 Auxiliary equation
m  0.2  0  m   0.2
So, complementary function
CF  C1e 0.2t
Particular integral
1 1
PI  10  10   e0t
D  0.2 D  0.2
10
PI   50
0  0.2
Solution x(t )  CF  PI
x(t )  50  C1e0.2t
As x(0)  1
So, t 0
x(0)  50  C1
1  50  C1
C1   49
 x(t )  50  49e0.2t
Hence, the correct option is (C).
Q.55 A 1 to 8 demultiplexer with data input Din , address inputs S0 , S1 , S2 (with S 0 as the LSB) and Y0 to Y7 as
the eight demultiplexed outputs, is to be designed using two 2 to 4 decoders (with enable E and address
inputs A0 and A1 ) as shown in the figure Din , S0 , S1 and S 2 are to be connected to P, Q, R and S but not
necessarily in this order. The respective input connections to P, Q, R and S terminals should be
PAGE
48

(A) S2 , Din , S0 , S1 (B) S1 , Din , S0 , S2

(C) Din , S0 , S1 , S2 (D) Din , S2 , S0 , S1


Ans. (D)
Sol. Consider a 1 8 demultiplexes

Output Y0 is given by
Y0  ( Din .S0 .S1.S2 )
Y0  ( Din  S0  S1  S2 )
Similarly Y1  ( Din  S0  S1  S2 )
Y2  ( Din  S0  S1  S2 )
Y3  ( Din  S0  S1  S2 )
Y4  ( Din  S0  S1  S2 )
Y5  ( Din  S0  S1  S2 )
Y6  ( Din  S0  S1  S2 )
Y7  ( Din  S0  S1  S2 )
For the circuit given in question, we can see that
Y0  (1A0  1A1  E )
Y0  ( R  S  P  Q)
Similarly Y1  (1A0  1A1  1E )
PAGE
49

Y1  ( P  Q  R  S )
and Y 4  (2 A0  2 A1  2E )  ( R  S  P  Q)
So comparing, we get
P  Din
Q  S2
R  S1
S  S0
Hence, the correct option is (D).


PAGE
1

.General Aptitude.

Q.1 to Q.5 carry one mark each


Q.1 Choose the correct verb to fill in the blank below :
Let us ______.
(A) introvert (B) alternate (C) atheist (D) altruist
Ans. (B)
Q.2 Choose the most appropriate word from the options given below to complete the following sentence.
If the athlete had wanted to come first in the race, he ______ several hours every day.
(A) should practice (B) should have practiced
(C) practised (D) should be practicing
Ans. (B)
Q.3 Find the missing sequence in the letter series below:
A, CD, GHI, ?, UVWXY
(A) LMN (B) MNO (C) MNOP (D) NOPQ
Ans. (C)
Q.4 If x > y > 1, which of the following must be true?
(i) ln x > ln y (ii) e x  e y (iii) y x  x y (iv) cos x  cos y
(A) (i) and (ii) (B) (i) and (iii) (C) (iii) and (iv) (D) (ii) and (iv)
Ans. (A)
Q.5 Choose the most suitable one word substitute for the following expression :
Connection of a road or way
(A) Perrinacious (B) Viaticum (C) Clandestine (D) Ravenous
Ans. (B)
Q.6 to Q.10 carry two marks each
Q.6 Ms. X will be Bagdogra from 01/05/2014 to 20/05/2014 and from 22/05/2014 to 31/05/2014. On the
morning of 21/05/2014, she will reach Kochi via Mumbai. Which one of the statements below is
logically valid and can be inferred from the above sentences?
(A) Ms. X will be in Kochi for one day, only in May.
(B) Ms. X will be in Kochi for only one day in May.
(C) Ms. X will be only in Kochi for one day in May.
(D) Only Ms. X will be in Kochi for one day in May.
Ans. (B)
PAGE
2

Q.7 log tan10  log tan 20   log tan 890 is _______.


1
(A) 1 (B) (C) 0 (D) –1
2
Ans. (C)
Q.8 In the following question, the first and the last sentence of the passage are in order and numbered 1 and
6. The rest of the passage is split into 4 parts and numbered as 2, 3, 4 and 5. These 4 parts are not
arranged in proper order. Read the sentences and arrange them in a logical sequence to make a passage
and choose the correct sequence from the given options.
1. On Diwali, the family rises early in the morning.
2. The whole family, including the young and the old enjoy doing this.
3. Children let off fireworks later in the night with their friends.
4. At sunset, the lamps are lit and the family performs various rituals.
5. Father, mother and children visit relatives and exchange gifts and sweets.
6. Houses looks so pretty with lighted lamps all around.
(A) 2, 5, 3, 4 (B) 5, 2, 4, 3 (C) 3, 5, 4, 2 (D) 4, 5, 2, 3
Ans. (B)
Q.9 Ram and Shyam shared a secret and promised to each other that it would remain between them. Ram
express himself in one of the following ways as given in the choices below. Identify the correct way as
per standard English.
(A) It would remain between you and me. (B) It would remain between I and you.
(C) It would remain between you and I. (D) It would remain with me.
Ans. (C)
Q.10 From a circular sheet of paper of radius 30 cm, a sector of 10% area is removed. If the remaining part is
used to make a conical surface, then the ratio of the radius and height of the cone is ______.
Ans. 2.064
PAGE
3

.Technical Part.

Q.1 to Q.25 carry one mark each


Q.1 In the circuit shown, diodes D1, D2 , and D3 are ideal, and the inputs E1 , E2 and E3 are ‘0 V’ for logic ‘0’
and ‘10 V’ for logic ‘1’. What logic gate does the circuit represent?

(A) 3-input OR gate (B) 3-input NOR gate (C) 3-input AND gate (D) 3-input XOR gate
Ans. (C)
Sol. Given circuit is shown below

If any of the input ( E1E2 E3 ) is ov (logic 0) Diode would conduct and output voltage V0 is 0 V (logic 0).
If all the inputs are 10 V (logic ‘1’)
None of the diode would conduct and output voltage V0 is 10 V (logic ‘1’)
Hence it is a logic of 3 input AND gate.
Hence the correct option is (C).
Q.2 The directivity of an antenna array can be increased by adding more antenna elements, as a larger
number of elements
(A) Improves the radiation efficiency
(B) Increases the effective area of the antenna
(C) Results in a better impedance matching
(D) Allows more power to be transmitted by the antenna
PAGE
4

Ans. (B)
Sol. Directivity (D) is increased by adding more antenna elements in an antenna array. Effective area ( Ae )
and directivity (D) are related by,
2
Ae  D
4
As directivity increases, aperture area also increases.
Hence the correct option is (B).
Q.3 The circuit shown consists of J-K flip-flops, each with an active low asynchronous reset ( Rd input). The
counter corresponding to this circuit is

(A) A modulo-5 binary up counter (B) A modulo-6 binary down counter


(C) A modulo-5 binary down counter (D) A modulo-6 binary up counter
Ans. (A)
Sol. Given counter is asynchronous counter with negative edge triggering.
Q is applied as negative edge triggering clock. Hence as negative edge triggering clock. Hence it is a up
counter.
Reset signal  Q2Q0
Counter reset at 101 hence it is a MOD – 5 counter.
Hence the correct option is (A).
Q.4 Consider the function g (t )  e t sin(2t )u (t ) where u (t ) is the unit step function. The area under g (t )
is ______.
Ans. 0.155
Sol. . Method 1 .
Given : g (t )  et sin(2t )u (t )
Area of the signal g (t ) is given by

A  g (t ) dt


e
t
A sin(2t ) u (t ) dt


A   et sin(2t ) dt
0
PAGE
5

Laplace transform of sin(2t ) is given by,



2
e
 st
sin (2t ) dt 

s  (2) 2
2

Putting s = 1, we get

2
e
 st
A sin (2t ) dt   0.155

1  (2) 2
2

Hence, the correct answer is 0.155.


. Method 2 .
Laplace transform of function g (t ) is given by

e
 st
G ( s)  g (t )dt


Putting s  0 , we get

G (0)   g (t )dt  Area of function g(t).

g (t )  et sin(2t )u (t )
Taking Laplace transform on both sides, we get
2
G( s) 
( s  1)  (2) 2
2

(Frequency shifting property of Laplace transform)


2
G (0)   0.155
1  (2) 2
2

Hence, the correct answer is 0.155.


Q.5 Which one of the following 8085 microprocessor programs correctly calculates the product of two 8-bit
numbers stored in registers B and C?
(A) MVI A, 00H (B) MVI A, 00H
JNZ LOOP CMP C
CMP C LOOP DCR B
LOOP DCR B JNZ LOOP
HLT HLT

(C) MVI A, 00H (D) MVI A, 00H


LOOP ADD C ADD C
DCR B JNZ LOOP
JNZ LOOP LOOP INR B
HLT HLT
PAGE
6

Ans. (C)
Sol. For option (C)
MVI A, 00 H ; Load 00 H to accumulator.
Loop ADD C ; Add the content of register C with accumulator.
DCR B ; Decrease the content of register B by 1.
INZ Loop ; Return to loop until contents of B becomes zero.
HLT,
In above program, content of register C add B times
i.e. [C] + [C] + [C] = B times = [C]  [B]
Hence, the correct option is (C).
Q.6 In the circuit shown, the voltage Vx (in Volts) is ______.

Ans. 0.8
Sol. Given circuit is shown below.

Applying KCL at node (1),


Vx Vx  0.25Vx
0.5    0.5Vx
20 10
Vx 0.75Vx 0.5Vx
0.5   
20 10 1
Vx  1.5Vx  10Vx
0.5 
20
12.5Vx  10
Vx  0.8 Volt
Hence, the correct answer is 0.8.
PAGE
7

Q.7 In the circuit shown, assume that diodes D1 and D2 are ideal. In the steady-state condition, the average
voltage Vab (in Volts) across the 0.5 F capacitor is ______.

Ans. 100
Sol. Circuit can be simplified as shown below

Above circuit is a voltage doubler hence in steady state condition, the average voltage Vab  2 Vm
Vab  2  50  100 Volt
Hence, the correct answer is 100.
Q.8 Which one of the following processes is preferred to form the gate dielectric (SiO 2 ) of MOSFETs?
(A) Sputtering (B) Molecular beam epitaxy
(C) Wet oxidation (D) Dry oxidation
Ans. (D)
Sol. The gate oxide thickness ( tox ) and hence the gate oxide capacitance ( Cox ) governs the electrical
properties of MOSFET. Hence its fabrication must be precised. For this dry oxidation technique is used.
Hence, the correct option is (D).
Q.9 The impulse response of an LTI system can be obtained by
(A) Differentiating the unit ramp response (B) Differentiating the unit step response
(C) Integrating the unit ramp response (D) Integrating the unit step response
Ans. (B)
Sol. For any LTI system,
d
(Impulse Response)  (step response).
dt
Hence, the correct option is (B).
PAGE
8

Q.10 Consider the bode plot shown in the figure. Assume that all the poles and zeros are real-valued. The
value of f H  f L (in Hz) is _______.

Ans. 8970
Sol. The given Bode plot is shown below.

Calculation of f L :
40  0
40 
log10 (300)  log10 ( f L )
 300 
log10   1
 L 
f
300  10 f L
f L  30 Hz ….. (i)
Calculation of f H :
0  40
 40 
log10 f H  log (900)
 f 
log10  H   1
 900 
f H  900 10  9000 Hz …… (ii)
f H  f L  9000  30  8970 Hz
Hence, the correct answer is 8970 Hz.
Q.11 The modulation scheme commonly used for transmission from GSM mobile terminals is
(A) 4-QAM (B) 16-PSK
(C) Walsh-Hadamard orthogonal codes (D) Gaussian Minimum Shift Keying (GMSK)
Ans. (D)
PAGE
9

Q.12 In the circuit shown using an ideal opamp, the 3-dB cut-off frequency (in Hz) is _____.

Ans. 159.15
Sol. . Method 1 :
Given circuit is shown below.

Circuit can be redrawn as

From non-inverting terminal : Change the circuit in Laplace domain

Voltage Va is given by,


1
Cs Vi ( s)
Va  Vi ( s) 
R
1 1  RCs
Cs
Using nodal analysis at inverting terminal
Va Va  V0 ( s )
 0
R R
2Vi ( s)
V0 ( s )  2Va 
1  RCs
PAGE
10

Voltage gain is given by,


V (s) 2
A( s )  0 
Vi ( s ) 1  RCs
Putting s  j , we get
2
A( j) 
1  jRC
2
A( j) 
1  (RC )2
At   0 , A( j 0)  A0  2
A0
At 3 dB cutoff frequency A( jC ) 
2
2 A0 2
 
1  (C RC ) 2
2 2
4
2
1  (C RC ) 2
2  1  (C RC ) 2
1  (C RC)2
C RC   1
1
C 
RC
1 1
fC    159.2 Hz
2RC 2104 107
Hence, the correct answer is 159.15.
. Method 2 :
Circuit can be redrawn as follows

1 1
For LPF, fH  
2RC 2(10 k) (0.1 F)
f H  159.15 Hz
Hence, the correct answer is 159.15.
PAGE
11

 n
1
Q.13 The value of  n   is ______.
n 0  2 

Ans. 2
Sol. z-transform of any signal x  n  is given by,

X ( z)   x  n z
n 
n

n
1
Let x  n  n   u  n
2
 n
1
Then X ( z)   n   u  n z n
n   2 

1 1
 n z
 
1
   
n
n   2 
u n z n
 2
2
 1 1 
1  z 
 2 
az 1 az
na nu  n  
Z .T .
 1 2

(1  az ) ( z  a)2
Putting z = 1, we get
 n
1 1/ 2
 n   u  n 
n   2  1/ 4
 n
1
Therefore  n   u  n  2
n   2 

Hence, the correct answer is 2.


Q.14 If C is a circle of radius r with centre z0 , in the complex z -plane and if n is a non-zero integer, then
dz
 C ( z  z ) n 1
0
equals

j
(A) 2nj (B) 0 (C) (D) 2n
2
Ans. (B)
dz
Sol. Given : I  (z  z )
C 0
n 1

where curve C is the circle of radius r with centre z0 in z-plane i.e.


C : z  z0  0
PAGE
12

Using Cauchy’s integral formula


f ( z ) dz
C ( z  z0 )n1  2i  f ( z0 )
n

where f ( z ) is analytic function within and on the boundary of curve ‘C’ and z0 is a point inside ‘C’.
So, f ( z )  1, f '( z )  0,
f "( z )  0,.........., f n ( z )  0
dz
  (z  z )
C 0
n 1
 j 2 0  0

Hence, the correct option is (B).


Q.15 The transfer function of a first order controller is given as
K (s  a)
Gc ( s ) 
sb
Where K , a and b are positive real numbers. The condition for this controller to act as a phase lead
compensator is
(A) a  b (B) a  b (C) K  ab (D) K  ab
Ans. (A)
Sol. For phase lead compensator, pole-zero diagram is shown below.

Hence, the correct option is (A).


Q.16 The contour on the x -y plane, where the partial derivative of x 2  y 2 with respect to y is equal to the
partial derivative of 6 y  4 x with respect to x, is
(A) y  2 (B) x  2 (C) x  y  4 (D) x  y  0
Ans. (A)
Sol. The partial derivative of x 2  y 2 w.r.t. y is
 2
(x  y2 )  2 y ……. (i) [Taking y terms as constant]
y
PAGE
13

The partial derivatives of 6 y  4 x w.r.t. x is



(6 y  4 x)  4 ……. (ii) [Taking x terms as constant]
x
From equation (i) and (ii), we have
So, 2y  4
y2
i.e. along the straight line.
Hence, the correct option is (A).
Q.17 For the circuit shown in the figure, the Thevenin’s equivalent voltage (in Volts) across terminals a  b is
______.

Ans. 10
Sol.
Applying KVL in outer loop
12  3(1  I )  6 I  0
12  3  3I  6I  0
9I  15
15 5
I   Amp
9 3
5
Hence, Vth  6 I  6   10 Volt
3
Hence, the correct answer is 10.
Q.18 At very high frequencies, the peak output voltage V0 (in Volts) is ______.

Ans. 0.5
Sol. At very high frequency i.e.    .
1
XC   0  capacitor acts as a short circuit
C
PAGE
14

1
V0  1.0sin(t )
11
1
V0  1.0sin(t ) 
2
So peak of output voltage = 0.5 V.
Hence, the correct answer is 0.5.
 1 tan x 
Q.19 For A    , the determinant of AT A1 is
  tan x 1 
2
(A) sec x (B) cos4x (C) 1 (D) 0
Ans. (A)
 1 tan x 
Given : Matrix A  
1 
Sol.
 tan x
 1  tan x 
AT  
 tan x 1 
A  1  tan 2 x  sec2 x
1 AT 1  1  tan x 
A   
A sec2 x  tan x 1 
 1  tan x  1  1  tan x 
AT A1    
 tan x
2
1  sec x  tan x 1 
1 1  tan 2 x 2 tan 2 x 
AT A1   
sec2 x  2 tan x 1  tan 2 x 
1
AT A1  (1  tan 2 x) 2  4 tan 2 x 
sec2 x 
1
AT A1  2
1  tan 4 x  tan 2 x  4 tan 2 x 
sec x
1
AT A1  2
1  tan 4 x  2 tan 2 x 
sec x
1
AT A1  2
(1  tan 2 x) 2
sec x
1
AT A1  2
 (sec2 x) 2
sec x
T 1
A A  sec2 x
Hence, the correct option is (A).
PAGE
15

Q.20 In the circuit shown in the figure, the BJT has a current gain () of 50. For an emitter-base voltage
VEB  600 mV, the emitter-collector voltage VEC (in Volts) is ______.

Ans. 2
Q.21 A coaxial cable is made of two brass conductors. The spacing between the conductors is filled with
Teflon (r  2.1, tan   0). Which one of the following circuits can represent the lumped element model
of a small piece of this cable having length z ?

(A) (B)

(C) (D)

Ans. (B)
Q.22 A message signal m(t )  Am sin (2f mt ) is used to modulate the phase of a carrier Ac cos(2f ct ) to get the
modulated signal y (t )  Ac cos[ 2f ct  m(t )] . The bandwidth of y (t )
(A) Depends on Am but not on f m (B) Depends on f m but not on Am
(C) Depends on both Am and f m (D) Does not depends on Am or f m
Ans. (C)
Sol. Given: m(t )  Am sin(2 f mt ) and c(t )  Ac cos(2 f c t )
PAGE
16

y(t )  Ac cos[2 f c t  m(t )]


According to Carson’s rule, bandwidth of PM signal is given by,
BW  2(f  f m )
So, it depends upon f m .
In PM maximum frequency deviation for sinusoidal signal is given by,
fmax  k p Am fm
So, f max depends on Am .
We can conclude that bandwidth depends upon both f m and Am .
Hence, the correct option is (C).
Q.23 If the base width in a bipolar junction transistor is doubled, which one of the following statements will
be TRUE?
(A) Current gain will increase (B) Unity gain frequency will increase
(C) Emitter-base junction capacitance will increase (D) Early voltage will increase.
Ans. (D)
Sol. If the base width is increased :
1. There will be more recombination in base region, which reduce the current gain.
2. Unity gain frequency fT   f  , since  decreases, f T also decreases.
3. Emitter base junction capacitance doesn’t depend on base width.
4. Effect of base width modulation will reduce. Hence, early voltage will increase.
Hence, the correct option is (D).
3
Q.24 Consider a four point moving average filter defined by the equation y [ n]    i x [ n  i ].
i 0

The condition on the filter coefficient that results in a null at zero frequency is
(A) 1   2  0;  0  3 (B) 1   2  1; 0  3
(C) 0  3  0; 1   2 (D) 1   2  0;  0  3
Ans. (A)
3
Sol. Given : y  n   i x  n  i 
i 0

y  n  0 x  n  1 x  n  1 2 x  n  2  3 x  n  3
Taking z-transform on both sides, we get
Y ( z )   0 X ( z )  1 X ( z ) z 1  2 X ( z) z 2  3 X ( z) z 3
Transfer function H ( z ) is given by
Y ( z)
H ( z)    0  1 z 1   2 z 2   3 z 3
X ( z)
Null at zero frequency
PAGE
17

z   0  1   2   3
For option (a), when 1   2  0
Then 0  0  0  0  3
 0   3
Hence, the correct option is (A).
10
Q.25 The phase margin (in degree) of the system G ( s)  is ______.
s ( s  10)
Ans. 84.35
10
Sol. Given : G( s) 
s( s  10)
Magnitude can be written as,
10
G ( j) 
 2  100
Phase angle can be written as,

G ( j)   900  tan 1
10
Gain crossover frequency is the frequency at which magnitude of G ( j) is unity.
10
1
gc 2gc  100

2gc (2gc  100)  100

4gc  100 2gc  100  0

Put gc  x ,
2

x 2  100 x  100  0
x  0.99,  100
x cannot be negative
2gc  0.99

gc  1 rad/sec

 0.99 
G ( jgc )   900  tan 1     95.65
0

 10 
Phase margin is defined as,
P.M.  1800  G( jgc )

P.M.  1800  95.650  84.350


PAGE
18

Hence, the correct answer is 84.35.

Q.26 to Q.55 carry two marks each


Q.26 Consider the 3 m long lossless air-filled transmission line shown in the figure. It has a characteristic
impedance of 120 , is terminated by short circuit, and is excited with a frequency of 37.5 MHz.
What is the nature of the input impedance ( Zin ) ?

(A) Open (B) Short (C) Inductive (D) Capacitive


Ans. (D)
Sol. Given : Z 0  120 , Z L  0 
f  37.5 MHz  37.5 106 Hz, l  3 m
Wavelength is given by,
vp 3 108
  8 m
f 37.5 106
For a lossless transmission line input impedance is given by,
 Z  jZ 0 tan l  2
Z in  Z 0  L  ; 
 Z 0  jZ L tan l  
  2  
 0  j120 tan  8  3  
Z in  120   
 120   0 
 
 
 3 
Z in  j120 tan     j120 
 4 
Since Z in is negative, it suggests a capacitive behavior.
Hence, option (D) is correct.
d 2 x(t ) dx(t )
Q.27 Consider the differential equation 2
3  2 x(t )  0.
dt dt
10
Given x(0)  20 and x(1)  , where e  2.718, the value of x (2) is ______.
e
Ans. 0.86
Sol. Given : Differential equation :
PAGE
19

d 2 x(t ) dx(t )
2
3  2 x(t )  0
dt dt
and x(0)  20, x(1)  10 / e
d
Let D
dt
So, the differential equation,
( D 2  3D  2) x  0
Writing auxiliary equation,
m 2  3m  2  0
m  1,  2
Solution, x(t )  C1e  t  C2e 2t
Applying boundary condition and evaluating constants
At t  0, x(0)  20
x(0)  C1  C2  20
C1  C2  20 ……. (i)
10
For t  1, x (1) 
e
10
 C1e 1  C2 e 2
e
C1e  C2  10e …… (ii)
By solving equation (i) and (ii),
10e  20
C1   4.18
e 1
C2  15.82
So, x(t )  4.18et  15.82e2t
For t  2, x(2)  4.18e2  15.82e4  0.86 Ans.
Hence, the correct answer is 0.86.
Q.28 The ABCD parameters of the following 2-port network are

 3.5  j 2 20.5   3.5  j 2 0.5 


(A) 
3.5  j 2 
(B) 
 20.5  0.5 3.5  j 2 
PAGE
20

 10 2  j0   7  j4 0.5 
(C) 
10 
(D)  
 2  j0  30.5 7  j 4 
Ans. (*)
Sol. Given circuit is shown below.

For above network, z-parameter is given by,


(5  j 4)  (2) 2 
 z  
 2 (5  j 4)  (2) 
7  j 4 2 
 z  
 2 7  j 4 
Hence, V1  (7  j 4) I1  2 I 2 …… (i)
V2  2 I1  (7  j 4) I 2 …… (ii)
ABCD parameter is given by,
V1  AV2  BI 2
I1  CV2  DI 2
From equation (ii),
2 I1  V2  (7  j 4) I 2
V2  V  j 4 
I1    I2 …… (iii)
2  2 
Put this in equation (i),
V  7  j 4  
V1  (7  j 4)  2    I2   2I2
2  2  
 7  j4  (7  j 4) (7  j 4)
V1    V2  I2  2I2
 2  2
 7  j4  (49  16)
V1    V2  I2  2I2
 2  2
V1  (3.5  j 2) V2  30.5I 2 …… (iv)
From equation (iii) and (iv),
 A B  3.5  j 2 30.5 
T   
C D   0.5 3.5  j 2 
Note : None of the options are correct.
Q.29 A network is described by the state model as x1  2 x1  x2  3u, x2   4 x2  u, y  3x1  2 x2 .
PAGE
21

 Y (s) 
The transfer function H ( s )    is
 U (s) 
11s  35 11s  35 11s  38 11s  38
(A) (B) (C) (D)
( s  2) ( s  4) ( s  2) ( s  4) ( s  2) ( s  4) ( s  2) ( s  4)
Ans. (A)
Sol. The state space model is
x1  2 x1  x2  3u
x2  4 x2  u
and y  3x1  2 x2
 x1   2 1  x1   3 
or  x    0 4   x    1 u
 2   2  
x 
y  [3 2]  1 
 x2 
Comparing with
x  Ax  Bu
y  Cx  Du
 2  1 3
A  , B    and C  3 2
 0 4   1
1 0   2 1  s  2 1 
 sI  A  s  
  
 
0 1   0 4   0 s  4 
1
s  2 1 
( s )  [ sI  A]  
s  4 
and
 0
1  s  4 1 
( s ) 
( s  2)( s  4)  0 s  2 
3
B 
 1
1 3( s  4)  1
( s ) B 
( s  2)( s  4)  0  ( s  2) 
So,

C  3, 2 
1  3s  13 
( s ) B 
( s  2)( s  4)  ( s  2) 
Transfer function is given by,
G ( s )  C ( s) B  D
PAGE
22

1
 [3(3s  13)  2( s  2)]
( s  2)( s  4)
11s  35

( s  4)( s  2)
Hence, the correct option is (A)
Q.30 Consider a continuous-time signal defined as
  t  
 sin  2   
  *
t   n
x(t )    (t  10 n)
   
  2 
   
Where ‘*’ denotes the convolution operation and t is in seconds. The Nyquist sampling rate (in
sample/sec) for x (t ) is ______.
Ans. 0.4
  t  
 sin  2   
  *
Sol. Given : x(t )  
  t   n 
  (t  10 n)
  2 
   
x(t )  x1 (t )  x2 (t )
 t 
sin  
x1 (t )   2   sin c  t 
Where  
 t  2
 
 2

And x2 (t )   (t  10n)
n 

 X ( f )  X1 ( f )  X 2 ( f )
( Convolution in time gives multiplication in frequency.)
  t   f 
X 1 ( f )  F sin c     2rect(2 f )  2rect  
  2   0.5 

  
and X 2 ( f )  F   (t  10n) 
 n  
 
1  m
    f    0.1  ( f  0.1m)
10 m   10  m 
PAGE
23

Multiplying X1 ( f ) and X 2 ( f ), we will get only two frequency. Components present in X ( f ) 0.1 Hz
and 0.2 Hz.
 BW of X ( f )  0.2 Hz
 Nyquist sampling rate is given by,
f s  2  BW  0.4 Hz
Hence, the correct answer is 0.4.
Q.31 A realization of a stable discrete time system is shown in the figure. If the system is excited by a unit
step sequence input x [n], the response y [ n] is

n n n n
 1  2  2  1
(A) 4    u [n]  5    u [n] (B) 5    u [n]  3    u [n]
 3  3  3  3
n n n n
1  2 2  1
(C) 5   u [n]  5   u [n] (D) 5   u [ n]  5   u [ n]
 3  3 3  3
Ans. (C)
Sol. The realization is done in direct form-II and the input output relationship can be written as.
2 5 5
y[n]  y[n  1]  y[n  2]   x[n  1]  x[n  2]
9 3 3
Taking z-transform on both sides, we get
2 5 5
Y ( z )  z 1Y [ z ]  z 2Y [ z ]   z 1 X [ z ]  z 2 X [ z ]
9 3 3
 2   5 5 
Y [ z ] 1  z 1  z 2   X ( z )   z 1  z 2 
 9   3 3 
Impulse response H ( z ) is given by
PAGE
24

Y ( z)
H ( z) 
X ( z)
 5 1 5 2 
 z  z 
H ( z)   
3 3
2
1  z 1  z 2
9
5 (1  z 1 )
H ( z )   z 1
3  1 1  2 1 
1  z 1  z 
 3  3 
If x[n]  u[n]
1
Then X [ z] 
1  z 1
5
 z 1 (1  z 1 )
Y ( z)  3
 1 1  2 1  1
1  z 1  z  (1  z )
 3  3 
5
 z 1
Y ( z)  3
 1 1   2 1 
1  z  1  z 
 3  3 
5 5
Y ( z)  
 1 1   2 1 
1  z  1  z 
 3   3 
Since the system is stable, hence both the signals will be right sided.
Taking inverse z-transform on both sides, we get
n n
1 2
y[n]  5   u[n]  5   u[n]
 3 3
Hence, the correct option is (C).
Q.32 In the circuit shown, both the enhancement mode NMOS transistors have the following characteristics :
kn  nCox (W / L)  1 mA/V2 ; VTN  1 V. Assume that the channel length modulation parameter  is
zero and body is shorted to source. The minimum supply voltage VDD (in Volts) needed to ensure that
transistor M 1 operates in saturation mode of operation is ______.
PAGE
25

Ans. 3
Sol. Given circuit is shown below

For transistor M 1 :
VG1  2 V, VS1  0 V and VTN  1V
For lower transistor ( M 1 ) to work in saturation, VSD1 must satisfy the following condition
VDS 1  VGS 1  VTN
So, for minimum VDD
VDS 1  VGS 1  VTN
VDS1  2  1  1 V
VD1  1 V
Drain current of M 1 is given by,
I D1  K '(VGS1  VTH )2
1 mA
I D1   (2  1) 2  1 mA
V2
For transistor M 2 :
VD  VG  VDD
Hence, VDG  VD  VG  0 V
PAGE
26

VS 2  VD1  1 V
For upper transistor ( M 2 ) to work in saturation, VSD 2 must satisfy the following condition
VDS 2  VGS 2  VTN
VD 2  VS 2  VG 2  VS 2  VTN
VDD  VD1  VG 2  VD1  VTN
VDD  1  VDD  1  VTN
0  1 volt
So it will work in saturation region and same current will flow (since drain is shorted to gate in M 2 , so it
is in saturation region)
I D 2  I D1  K '(VGS 2  Vth )2
1 mA
1 mA  2
 (VDD  1  1) 2  VS 2  VD1 
V
 VDD  3 Volt
Hence, the correct answer is 3.
Q.33 The Newton-Raphson method is used to solve the equation f ( x)  x3  5 x 2  6 x  8  0. Taking the
initial guess as x  5, the solution obtained at the end of the first iteration is ______.
Ans. 4.2903
Sol. Given : The non-linear equation
f ( x)  x 3  5 x 2  6 x  8  0
With initial guess x0  5
Taking 1st derivative,
f '( x)  3x 2  10 x  6
Using N-R method, the (n  1)th iteration is
f ( xn )
xn 1  xn 
f '( xn )
For n  0
f ( x0 )
First iteration, x1  x0 
f '( x0 )
x03  5 x02  6 x0  8 53  5  52  6  5  8
x1  x0   5 
3 x02  10 x0  6 3  52  10  5  6
22
x1  5  4.2903 Ans.
31
Hence, the correct answer is 4.2903.
1 x
Q.34 The variance of the random variable X with probability density function f ( x)  x e is ______.
2
Ans. 6
PAGE
27

1 x
Sol. Given : Probability density function f ( x)  xe
2
Expected or mean value of X
 
1
  E ( X )   xf ( x) dx   x x e dx
x


2 
0 
1 1
 
2 
x( x)e (  x ) dx   x  x  e  x dx
20
[ x has two values of – x and x in different area]
0 
1 1
 
2 
x 2e x dx   x 2e  x dx
20
Let   I1  I 2

1
0
1
where I1  
2 
x 2e x dx and I 2   x 2e x dx
20
Now, for evaluation of I1
Let x  t  dx   dt
When x    to 0  t   to 0
Putting new limits
0 0
1 1
I1    x 2e x dx    (t ) 2 e  t (dt )
2  2
0 
1 1
I1   t 2et (dt )    t 2et dt
2 20

1 2 x  a a

I1  
2 0
x e dx 


0
f (t ) dt   f ( x) dx 
0 
So, I  I1  I 2
 
1 1
I    x 2e x dx   x 2e x dx  0
20 20
So   E( X )  0
Variance of X is
Var ( X )  2X  E ( X  )2 
i.e. expected value of ( x  ) 2
Var ( X )  E ( X 2 )
 
1
x x
x
 2
f ( x) dx  2
x e dx
 
2
0 
1 1
 
2 
x 2 ( x) e  (  x ) dx   x 2 ( x) e  x dx
20
PAGE
28

0 
1 1
 
2 
x3 e x dx   x3 e  x dx
20
 I3  I 4
0 
1 1
where I3   
2 
x3e x dx and I 4   x3e x dx
20
Now, for evaluation of I 3
Let x  t  dx   dt
When x    to 0  t   to 0
0 0
1 1
I 3    (t )3 e  t (dt )    t 3e  t dt
2 2
 
1 1
I 3   t 3et dt   x3e x dx  I 4
20 20
  
1 1
So, I   x3e x dx   x3e x dx   x3e  x dx
20 20 0

 
 e x 
I x    3x 2 ( e x )dx
3

  1 0 0


I   x3e x  3 x 2e x dx
0
0

 

 e x  
I    0  0   3  x 2 ( e  x )   2 x   dx 
  1  
0
0

I  6 xe x dx
0

 x 

 
I  6   xe   1(e )dx   6  e  x dx
x

 
0
0 0

I  6 e x   6 0 1  6 Ans.
0

Note : Standard derivation  x


ˆ x  Var ( X )  6
As Var ( X )  E ( X  )2
 E  X 2  2X  2   E ( X 2 )  2E ( X )  2 E (1)
 E ( X 2 )  2    2 1
Var ( X )  E ( X 2 )  2  E ( X 2 )   E ( X )
2
Or
Hence, the correct answer is 6.
 n 
Q.35 Let x [n]  1  cos   be a periodic signal with period 16. Its DFS coefficients are defined by
 8 
PAGE
29

1 15  n 
ak  
16 n0
x [n]exp   j kn  for all k. The value of the coefficient a31 is ______.
 8 
Ans. 0.5
 n 
Sol. Given : xˆ[n]  1  cos  
8 
Time period N  16 .
DFS (Discrete Fourier Series) is periodic and discrete in frequency domain.
For DFS ak  ak  N
a31  a1516  a15
and a15  a116  a1
 n 
xˆ[n]  1  cos  
8 
1  jn  j n

ˆx[n]  1  e 8  e 8 
2 
1  j8n 1 j n
xˆ[n]  e 1  e 8
2 2
1 1
a1  , a0  1 and a1 
2 2
1
Hence a31  a1 
2
Hence, the correct answer is 0.5.
Q.36 The position control of a DC servo-motor is given in the figure. The values of the parameters are
KT  1 N-mA, Ra  1, La  0.1H, J  5 kg-m2 , B  1 N-m(rad/sec) and Kb  1 V/(rad/sec).
The steady-state position response (in radians) due to unit impulse disturbance torque Td is ______.

Ans. – 0.5
Sol. Given : KT  1 N-m/A, Ra  1, La  0.1H, J  5 kg-m2 , B  1 N-m/(rad/sec) , Kb  1 V/(rad/sec)
PAGE
30

For unit impulse Td ( s )  1


1
X ( s)
 Js  B
 TD ( s) 1  1 K b KT
( Js  B) ( Ra  La s )
X (s) 1

 TD ( s ) ( Js  B ) ( Ra  La s )  K b KT
( s ) ( s ) X ( s )
 .
TD ( s ) X ( s ) TD ( s )
( s ) 1 X ( s)
 .
TD ( s ) s {TD ( s )}
( s) 1

 TD ( s) s  ( Js  B) ( Ra  La s)  Kb KT 
1
( s) 
s  ( Js  B) ( Ra  La s)  Kb KT 
Steady state response can be calculated using final value theorem.
Applying final value theorem,
1
 ss  lim s  ( s )  lim
s 0 s  0 ( Js  B ) ( R  L s )  K K
a a b T

1 1
 ss     0.5
BRa  K b KT 1  1
Hence, the correct answer is – 0.5.
Q.37 The characteristic equation of an LTI system is given by F ( s)  s 5  2s 4  3s 3  6s 2  4s  8  0.
The number of roots that lie strictly in the left half s-plane is __________.
Ans. 2
Sol. Given : s 5  2s 4  3s 3  6s 2  4s  8  0
Routh Tabulation :

s5 1 3 –4
s4 2 6 –8
s3 0 0 0
PAGE
31

4
Because a row of zeros appears, we form the auxiliary equation using the coefficients of s row
A( s)  2s 4  6s 2  8  0
The derivative of A(s) with respect to s is
d
 2 s 4  6 s 2  8  8s 3  12s
ds 
3
From which the coefficients 8 and 12 replace the zeros in the s row of the original tabulation. The
remaining portion of the Routh's tabulation is
s3 8 12 0
s2 3 –8 0
s1 33.33 0 0
s0 –8 0 0

Let x  s , then
2

2 x 2  6 x  8  x  1,  4
s 2  1,  4  s  1,  2 j
Number of roots lies on RHS is at s  1 .
Number of roots on the imaginary axis = 2
 Number of roots in LHS of s plane  5  (2  1)  2
Hence, the correct answer is 2.

Q.38 A random binary wave y (t ) is given by y (t )  X
n 
n p (t  nT  ) ,

Where p (t )  u (t )  u (t  T ), u (t ) is the unit step function and  is an independent random variable with
uniform distribution in [ 0, T ]. The sequence consists of independent and identically distributed binary
valued random variables with P [ X n  1]  P [ X n  1]  0.5 for each n.
 3T    3T 
The value of the auto correction Ryy    E  y (t ) y  t    equals ______.
 4    4 
Ans. 0.25
Sol. For a binary symmetric pulse
 
 RYY ()  1  
 T
 3T 
 
 3T 4  1
 RYY    1     0.25
 4  T  4
 
Hence, the correct answer is 0.25.
Q.39 A universal logic gate can implement any Boolean function by connecting sufficient number of them
appropriately. Three gates are shown.
PAGE
32

Which one of the following statements is TRUE?


(A) Gate 1 is a universal gate (B) Gate 2 is a universal gate
(C) Gate 3 is a universal gate (D) None of the gates shown is a universal gate
Ans. (C)
Sol. For a universal gate, it consist of one basic gate (AND, OR) and inverter (NOT gate).
Gate (1) and Gate (2) don’t have inverter.
Gate (3) is a gate with inverter.
Hence Gate (3) is a universal gate.
Hence, the correct option is (C).
Q.40 Two sequences x1[n] and x2 [n] have the same energy. Suppose x1[n]   0.5n u [n], where  is a
 1.5 for n  0,1

positive real number and u [ n] is the unit step sequence. Assume x2 [n]   . Then the

 0 other wise.
value of  is ______.
Ans. 1.5
Sol. Given : x1  n   0.5n u  n
 1.5 for n  0,1
And x2  n  
0 otherwise

x1  n   x  n
2
Energy of 1
n 
 

  0.5 u  n     (0.5)


2
 n 2 2n

n  n0

    (0.5)   (.5)4 
2 2 2 2

2 2 4 2
  
1  (0.5) 2 1  1 3
4

x2  n   x  n
2
Energy of 2
n 

 ( 1.5)2  ( 1.5)2  1.5  1.5


=3
Energy of x1  n and x2  n are equal
PAGE
33

4 2
Hence,  3
3
9
2 
4
3
   1.5
2
Hence, the correct answer is 1.5.
  t  
 sin  5    
Q.41 The complex envelope of the bandpass signal x(t )      sin  t   , centered about f  Hz,
1
 t   4 2
 
 5 
is
  t     t     t     t  
 sin  5   j   sin  5    j   sin  5   j   sin  5    j 
(A)    e 4 (B)    e 4 (C) 2     e 4 (D) 2     e 4
 t   t   t   t 
       
 5   5   5   5 
Ans. (C)
  t  
 sin  5    
Sol. Given : x(t )   2     sin  t  
  t    4
  5  

x (t ) can also be written as


x(t )  Re  xˆ (t )e j 2 fct  …… (i)
1
Where f c  Hz
2
and xˆ (t ) = Complex envelope
Also, sin   Re  je j 

  t  
 sin  5    j  t   
 

x(t )   2     Re   je  4

  t    
  5  

   t   
  sin    j  t      
 5  
x(t )  Re  2     e  4 2  

   t   
    
  5   
PAGE
34

   t   
  sin     
 j 
x(t )   2    e 4   e jt 
5
   t   
    
  5 
 
Comparing with equation (i), we get
  t  
 sin  5   j
xˆ (t )  2     e 4
  t  
  5  
Hence, the correct option is (C).
Q.42 A 200 m long transmission line having parameters shown in the figure is terminated into a load RL . The
line is connected to a 400 V source having source resistance RS through a switch, which is closed at
t  0. The transient response of the circuit at the input of the line ( z  0) is also drawn in the figure. The
value of RL (in  ) is ______.

Ans. 30
Sol. Given : Rs  150 , Vs  400 V, R0  50 

At 2 s , the input voltage at transmission line becomes 62.5 V.


Using voltage bounce diagram,
PAGE
35

From above figure,


V0   LV0   L  sV0  62.5 volts …… (i)
V0  100 V
RL  R0 RL  50 Rs  R0 150  50
L   , s    0.5
RL  R0 RL  50 Rs  R0 150  50
Substituting these values in equation (i), we get
100(1   L  0.5 L )  62.5
 L   0.25
RL  50
  0.25
RL  50
RL  30 
Hence, the correct answer is 30.
Q.43 A vector field D  22 a  za z exists inside a cylindrical region enclosed by the surfaces   1, z  0
and z  5. Let S be the surface bounding this cylindrical region. The surface integral of this field on
S  D  ds is ______.
S

Ans. (78.54)
Sol. Vector field D  22 aˆ  zaˆ z in the region enclosed by the surfaces
  1, z  0 & z  5
S be the surface boundary of the region so surface integral on S is given by,
I  D  ds
s
PAGE
36

S is the closed surface of cylindrical.


Using Gauss Divergence theorem
I   D  ds   (  D)dv
v
s

Divergence of D i.e.
1  D  1 D Dz
div D    D    
      z
D  2 , D  0 & Dz  z
2

1  
So, D  (  22 )  0   z
  z
2.3 2
  1  6  1

V is the volume bounded by closed surface ‘S’
dv  d (d )dz  d d dz
z 5 2  1
I    (6  1) d d dz
z  0  0  0

z 5 2 
1 
I z 0 0 0
     d dz
2
(6 ) d
 
z 5 2  1
2
I   2  d dz
3

z  0  0
2 0
z 5 2 
 1
I    2 1  2  d  dz
z  0  0

5  
z 5 2 
    d  dz
2 z 0  0 
5
  2 5  25  78.5
2
Note : The volume has been covered in this manner z  0 to z  5 i.e. height of the cylinder, the base
area is (d )  d 
PAGE
37

Hence, the correct answer is 78.5.


Q.44 In the circuit shown, assume that the opamp is ideal. If the gain V0Vm  is  12, the value of R (in k )
is ______.

Ans. (1)
Q.45 Suppose x [ n] is an absolutely summable discrete-time signal. Its z-transform is a rational function with
two poles and two zeroes. The poles are at z  2 j. Which one of the following statements is TRUE for
the single x [ n] ?
(A) It is a finite duration signal (B) It is a causal signal
(C) It is a non-causal signal (D) It is a periodic signal
Ans. (C)
Sol. Poles are at z   2 j
Which are outside the unit circle, since x  n  is absolutely summable, therefore ROC must include the
unit circle, as shown below.

ROC is inside the unit circle, therefore it is a non-causal signal.


Hence, the correct option is (C).
PAGE
38

Q.46 The electric field profile in the depletion region of a p -n junction in equilibrium is shown in the figure.
Which one of the following statements is NOT TRUE?

(A) The left side of the junction is n - type and the right side is p- type
(B) Both the n - type and p- type depletion regions are uniformly doped
(C) The potential difference across the depletion region is 700 mV
15 3
(D) If the p- type region has a doping concentration of 10 cm , then the doping concentration in the
16 3
n - type region will be 10 cm .
Ans. (C)
Sol. Given : Emax  104 V/cm  106 V/m
W  1  (0.1) m  1.110 6 m
From poisson’s equation
d 2V  V

dx 2 
 dV
E
dx
dE V

dx 
V
E   dx

E   V
Electric field for p-n junction diode is given by,
PAGE
39

Electric field for n-p junction is given by,

So, given electric field profile is of n-p junction.


Option (A) satisfies that left side of junction is n-type and right side is p-type.
Option (B) Also satisfies because V  constant. i.e. uniformly doped.
Option (C)
potential difference V is given by,
PAGE
40

V    E  dx
V = – (Area under E-x curve)
1
V   1.110 4 104
2
V   550 mV
So, option (C) is wrong.
Option (D)
for given profile
 p  n

N A  N D

1015  1016
So, option (D) is also correct.
Hence, the correct option is (C).
Q.47 A coaxial capacitor of inner radius 1 mm and outer radius 5 mm has a capacitance per unit length of 172
pF/m. If the ratio of outer radius to inner radius is doubled, the capacitance per unit length (in pF/m) is
______.
Ans. 120.22
Sol. .Method 1.
Given : Inner radius a = 1 mm  103 m
Outer radius b = 5 mm  5 103 m

C  172 pF/m  172 1012 F/m


Capacitance per unit length is given by,
2
C ;   permittivity of medium
b
ln  
a
2
172 1012 
 5 103 
ln  3 
 10 
PAGE
41

172 1012  5 103 


 ln  3 
2  10 
  4.405 1011 F/m
b
Now as per the question, the ratio is doubled,
a
b
   5  2  10
 a new
2 4.405 1011
Then Cnew   1.202 1010 F/m
ln (10)
Cnew  120.2 pF/m
Hence, the correct answer is 120.2.
.Method 2.
1
C
b
ln  
a
b
ln  
  1
C2 a
C1 b
ln  
 a 2
Given, C1  172 pF/m
b b
   5,    10
 a 1  a 2
172 ln (5)
 C2  pF/m =120.2 pF/m
ln (10)
Hence, the correct answer is 120.2.
Q.48 The current in an enhancement mode NMOS transistor biased in saturation mode was measured to be 1
mA at a drain-source voltage of 5 V. When the drain-source voltage was increased to 6 V while keeping
gate-source voltage same, the drain current increased to 1.02 mA. Assume that drain source to saturation
voltages is much smaller than the applied drain-source voltage. The channel length modulation
1
parameter  (in V ) is ____.
Ans. 0.022
Sol. Given : In saturation mode in NMOS, the drain current I D  1 mA and drain to source voltage
VDS  5 V for same VGS if VDS  6 V then I D  1.02 mA .
In saturation mode
I D  I DS (1  VDS )
Where I DS  Drain current in saturation.
PAGE
42

For VDS  5 V and I D  1 mA


1  I D (1  5) …… (i)
For VDS  6 V and I D  1.02 mA
1.02  I DS (1  6) …… (ii)
By dividing equation (i) and (ii)
1 1  5

1.02 1  6
  0.022 V 1
Hence, the correct answer is 0.022.
Q.49 For the system shown in the figure, s   2.75 lies on the root locus if K is ______.

Ans. 0.3
Sol. . Method 1 :
10 K ( s  3) K '( s  3)
G( s) H ( s)  
( s  2) ( s  2)
Product of vector lengths drawn from the poles of G ( s ) H ( s ) to s
For K ' : K'
Product of vector lengths drawn from the zeros of G ( s ) H ( s ) to s
0.75
K' 3
0.25
10K  3
K  0.3
Hence, the correct answer is 0.3.
. Method 2 :
By magnitude criteria,
G( s) H ( s) s 2.75  1
K 10 2.75  3
1
2.75  2
0.75  1
K  0.3
0.25  10
Hence, the correct answer is 0.3.
Q.50 A three bit pseudo random number generator is shown. Initially the value of output Y  Y2Y1Y0 is set to
111. The value of output Y after three clock cycles is
PAGE
43

(A) 000 (B) 001 (C) 010 (D) 100


Ans. (D)
Sol. Given circuit is equivalent to

Truth table for Y2Y1Y0 is shown below


Y2 Y1 Y0
Initial 1 1 1
st
1 clock 0 1 1
nd
2 clock 0 0 1
3rd clock 1 0 0
Hence after 3 clock pulse Y is 100.
Hence, the correct option is (D).
Q.51 In the circuit shown, the current I flowing through the 50 resistor will be zero if the value of capacitor
C (in s ) is ______.

Ans. 20
Sol.

Given : Supply voltage Vs  5 sin 5000t


So frequency   5000 rad/sec
PAGE
44

For current I  0 the impedance Z   (open circuit) or Y  0

1 1
Y 
jL jL  1
jC
1 jC
Y 
jL 1  2 LC
 C 1 
Y  j 
1   LC L 
2

 2 LC  1  2 LC 
Y  j 
 L(1   LC ) 
2

 22 LC  1 
Y  j 
 L(1   LC ) 
2

When Y  0 then 2 LC  1  0
2

1
C 2
2 L
1
C  20 F
2  (5000) 2 1103
Hence, the correct answer is 20.
Q.52 A fair dice with faces [1, 2, 3, 4, 5, 6] is thrown repeatedly till ‘3’ is observed for the first time. Let X
denote the number of times the dice is thrown. The expected value of X is ______.
Ans. (6)
1
Sol. We have probability of getting 3 
6
1 5
Probability of not getting 3  1 

6 6
Now, the random variable X represents the number of throws required for getting 3.
1
So, X  1, P( x  1) 
6
5 1
X  2, P( x  2)  
6 6
5 5 1
X  3, P( x  3)   
6 6 6
PAGE
45

5 5 5 1
X  4, P( x  4)    
6 6 6 6
For discrete random, the mean value is given by,
m
E[ X ]   xi P ( xi )
i 1

2 3
1  5  1  5 1 5 1
X  (1)    (2)     (3)      (4)      ..........
6  6  6  6 6 6 6
1 
2 3
5 5 5
X    1  (2)    (3)    (4)    ........
 6   6 6 6 
2 3
5 5 5
Let S  1  (2)    (3)    (4)   …(i)
6 6 6
2 3 4
5 5 5 5 5
  S   2    3   4   …(ii)
6 6 6 6 6
Subtracting equation (ii) from (i), we get
2 3
5 5 5 5
S    S  1  (2  1)    (3  2)    (4  3)  .......
6 6 6 6
2 3
S 5 5 5
 1         .......
6 6 6 6
S 1
  6  S  36
6 1 5
6
The expected value of X is
36
E[ X ]  6
6
Hence, the correct answer is 6.
Q.53 In the circuit shown, assume that the diodes D1 and D2 are ideal. The average value of voltage Vab (in
Volts), across terminals ‘a’ and ‘b’ is ______.

Ans. 5
Sol. During positive half cycle
PAGE
46

Diode D1 is ON and D2 is OFF.

10
Vab   6 sin t
10  20
Vab  2 sin t
During negative half cycle
Diode D1 is OFF and D2 is ON.

6 sin t
Vab   3 sin t
2
 2 sin t 0  t  
Hence Vab  
3 sin t   t  2
Average value of Vab is given by
T
1
T 0
(Vab )avg  Vab d (t )
2
1
2 0
(Vab )avg  Vab d (t )

1  2
(Vab ) avg    2 sin t d (t )   (3 sin t ) d (t )
 2 0 

1 3 
2   cos t 0   cos t 
 2
(Vab ) avg 
2 2
3
(Vab ) avg    cos   cos0  
2
cos 2  cos 
3
(Vab )avg  (1  1)  (1  1)
2
(Vab ) avg  5 Volt
Hence, the correct answer is 5.
PAGE
47

Q.54 An npn BJT I having reverse saturation current I S  1015 A is biased in the forward active region with
VBE  700 mV. The thermal voltage (VT ) is 25 mV and the current gain () may vary from 50 to 150
due to manufacturing variations. The maximum emitter current (in μA ) is ______.
Ans. 1475
Sol. Given : I S  10 15 A
VBE  700 mV
VT  25 mV
  50 to 150
Collector current of BJT is given by,
VBE

IC  I S e VT
   1
(  1)
And IE  IC

V
(  1) BE

IE  I S e VT

I E Will be maximum when  is 50.
700103
(50  1)
 1015 e 2510
3
( I E ) max
50
( I E )max  1475 A
Hence, the correct answer is 1475.
Q.55 An SR latch is implemented using TTL gates as shown in the figure. The set and reset pulse inputs are
provided using the push-button switches. It is observed that the circuit fails to work as desired. The SR
latch can be made functional by changing

(A) NOR gates to NAND gates


(B) Inverters to buffers
(C) NOR gates to NAND gates and inverters to buffers
(D) 5 V to ground
Ans. (D)
PAGE
48

Sol. For the given circuit, if switch is pushed the input is +5V (i.e. logic1)
But it the switch is not pushed the input is floating. In TTL logic a floating input is considered as logic
1.
Hence input is logic-1 for both the states of switch. The desired operation can be achieved by changing
5V to GND; so that when switch is ON, input is logic-0 and when switch is OFF input is logic-1.
Hence, the correct option is (D).



You might also like